Download as pdf or txt
Download as pdf or txt
You are on page 1of 158

Hand and Wrist Scored and Recorded Self-Assessment Examination 2019

AAOS
Question 1 of 100

The pathology of the lesion shown in Figures 1 and 2 reveal what cellular pattern?

Figure 1 Figure 2

A. Uniform distribution of stromal cells and giant cells


B. Mixture of mature fat cells and spindle cells
C. Mucin-filled space with occasional spindled fibroblasts
D. Lobular pattern of vascular proliferation with inflammation

Correct Answer: C

Discussion

The MRIs display the classic appearance of a ganglion cyst, which is a benign fluid-filled sac
most commonly presenting at the wrist level. The signal characteristics of a ganglion on MRI are
low signal on the T1-weighted image and high signal on the T2-weighted image, as revealed in
Figure 1 and 2. The pathology of a ganglion cyst is characterized by a mucin-filled synovial
lined sac with spindled fibroblasts.

A uniform distribution of stromal cells and giant cells would fit the pathologic description of a
giant cell tumor of the tendon sheath, and numerous giant cells are the hallmark of this lesion.
This type of tumor is benign, usually presenting in the region of the digital flexor sheath.
Recurrence after excision is common. MRI of giant cell tumors shows an extra-articular soft-
tissue mass, and on both T1-weighted and T2-weighted images, some portions of the tumor show
decreased signal intensity.

A mixture of mature fat cells and spindle cells would fit the pathologic description of a spindle
cell lipoma. This benign tumor of mature fat usually is found in a superficial/subcutaneous
location. The pathology is characterized by a mixture of mature fat cells and spindle cells. MRI
would show high signal intensity on both T1-weighted and T2-weighted images.

A lobular pattern of vascular proliferation with inflammation would fit the pathologic description
of a pyogenic granuloma. These skin growths are small, round, and usually blood red in color.
They frequently bleed, because they contain a large number of blood vessels. The pathology is
characterized by a lobular pattern of vascular proliferation with inflammation. MRI findings
show isointensity on the T2-weighted image. These MRI findings are consistent with
hypervascularity.

Recommended Readings

Dinauer PA, Brixey CJ, Moncur JT, Fanburg-Smith JC, Murphey MD. Pathologic and MR imaging
features of benign fibrous soft-tissue tumors in adults. Radiographics. 2007 Jan-Feb;27(1):173-87.
PubMed

Payne WT, Merrell G. Benign bony and soft tissue tumors of the hand. J Hand Surg Am. 2010
Nov;35(11):1901-10. doi: 10.1016/j.jhsa.2010.08.015. Epub 2010 Oct 20. PubMed

Nahra ME, Bucchieri JS. Ganglion cysts and other tumor related conditions of the hand and wrist. Hand
Clin. 2004 Aug;20(3):249-60, v. PubMed

Question 2 of 100

Figures 1 and 2 are the radiographs of a 35-year-old right-hand-dominant man who has had progressive
right wrist pain for 1 year. There is no history of trauma, and he has had no treatment to date. He reports
some pain at rest with limited motion and substantial pain with use. He is currently out of work on short-
term disability because of this wrist problem. An examination reveals mild dorsal wrist swelling,
decrease wrist range of motion, and decreased grip strength. Contralateral wrist examination findings are
normal. What is the most appropriate course of treatment?
Figure 1 Figure 2

A. Proximal row carpectomy


B. Radial shortening osteotomy and vascularized bone grafting
C. Scaphoid excision and midcarpal arthrodesis
D. Capitate hamate fusion

Correct answer: A

 Discussion

This patient has late-stage Kienböck disease. According to the Lichtman classification for Kienböck
disease, this would represent stage IIIB, with lunate collapse/fragmentation, loss of carpal height
secondary to proximal capitate migration, and a flexed scaphoid. The lateral radiograph reveals a
radioscaphoid angle exceeding 60 degrees, so disease stage is IIIB. According to Condit and associates,
when the presurgical radioscaphoid angle exceeds 60 degrees, results are poor when an attempt to
maintain the lunate is made. As a result, the procedure with the most predictable outcome is a proximal
row carpectomy. A radial-shortening osteotomy could be performed because the ulnar variance is
negative. Considering the marked lunate fragmentation and collapse, a vascularized bone graft likely is
contraindicated and associated with less predictable results than a proximal row carpectomy. There is no
role for supervised hand therapy and splinting in the setting of advanced Kienböck disease. Similarly,
there is no role for maintenance of the lunate in the setting of advanced collapse and fragmentation.

 Recommended Readings

Nakamura R, Horii E, Watanabe K, Nakao E, Kato H, Tsunoda K. Proximal row carpectomy versus
limited wrist arthrodesis for advanced Kienböck's disease. J Hand Surg Br. 1998 Dec;23(6):741-5.
PubMed

Lichtman DM, Mack GR, MacDonald RI, Gunther SF, Wilson JN. Kienböck's disease: the role of
silicone replacement arthroplasty. J Bone Joint Surg Am. 1977 Oct;59(7):899-908. PubMed

Goldfarb CA, Hsu J, Gelberman RH, Boyer MI. The Lichtman classification for Kienböck's disease: an
assessment of reliability. J Hand Surg Am. 2003 Jan;28(1):74-80. PubMed

Condit DP, Idler RS, Fischer TJ, Hastings H 2nd. Preoperative factors and outcome after lunate
decompression for Kienböck's disease. J Hand Surg Am. 1993 Jul;18(4):691-6. PubMed

Question 3 of 100

Assuming that the fracture shown in this radiograph (Figure 1) is aligned on the anteroposterior radiograph and
heals in this position, secondary to fracture malalignment, there will be loss of active

Figure 1

A. metacarpophalangeal (MP) joint extension.


B. proximal interphalangeal (PIP) joint extension.
C. MP flexion.
D. PIP joint flexion.

Correct answer: B

 Discussion

This is a transverse proximal phalanx fracture with apex volar angulation. The fracture displaces into an
apex volar angulated position under the pull of the central slip on the distal fragment and the interossei
insertions at the base of proximal phalanx. Although it is possible to lose motion in flexion or extension
of the MP or PIP joints, the biomechanics will not allow full extension of the PIP joint. If allowed to
heal in apex palmar malunion, the predicted corresponding extensor lags are for a 10-degree lag at 16
degrees of angular deformity, a 24-degree lag at 27 degrees of deformity, and a 66-degree lag at 46
degrees of deformity. These fractures usually can be treated with closed reduction with or without
percutaneous pinning. With surgical treatment, there may be loss of motion both at the MP and PIP
joints.

 Recommended Readings

Faruqui S, Stern PJ, Kiefhaber TR. Percutaneous pinning of fractures in the proximal third of the
proximal phalanx: complications and outcomes. J Hand Surg Am. 2012 Jul;37(7):1342-8. doi:
10.1016/j.jhsa.2012.04.019. PubMed

Henry MH. Fractures of the proximal phalanx and metacarpals in the hand: preferred methods of
stabilization. J Am Acad Orthop Surg. 2008 Oct;16(10):586-95. Review. Full text

Question 4 of 100

Figures 1 through 3 are the radiographs of a 27-year-old man who has had wrist pain since falling 1 day ago.
Which treatment offers the best prognosis for prevention of carpal collapse and progressive arthritis?
Figure 1 Figure 2 Figure 3

A. Long-arm cast
B. Percutaneous screw fixation
C. Open reduction and internal fixation (ORIF) with bone graft
D. Proximal row carpectomy

Correct answer: C

Discussion

Although this patient’s history includes a recent fall, the radiographs show evidence of a scaphoid
nonunion with carpal collapse but no arthritis. Obtaining union of the scaphoid is important to prevent
progressive carpal collapse and arthritic changes. ORIF with bone graft is most appropriate to obtain
union and correct the collapse deformity. Screw fixation with volar wedge graft often is performed to
realign a scaphoid humpback deformity, although cancellous bone graft also is a reasonable option.
Vascularized bone graft is considered for a nonunion of long duration, avascular necrosis of the
proximal pole, and failed prior surgery. Cast immobilization will not lead to union of the scaphoid.
Percutaneous screw fixation is not indicated for the treatment of a displaced nonunion. A proximal row
carpectomy is a salvage procedure and is not indicated for this patient because there are no arthritic
changes.

 Recommended Readings
Kawamura K, Chung KC. Treatment of scaphoid fractures and nonunions. J Hand Surg Am. 2008 Jul-
Aug;33(6):988-97. doi: 10.1016/j.jhsa.2008.04.026. Review. PubMed

Moon ES, Dy CJ, Derman P, Vance MC, Carlson MG. Management of nonunion following surgical
management of scaphoid fractures: current concepts. J Am Acad Orthop Surg. 2013 Sep;21(9):548-57.
doi: 10.5435/JAAOS-21-09-548. Review. Full text

Question 5 of 100

Figures 1 and 2 show the clinical photograph and ultrasonography image obtained from an 8-month-old boy
who has a 2-month history of a well-circumscribed mass in the palm, just proximal to the palmar digital crease of
the index finger. The mass has not changed in size and does not seem to cause pain. What is the best next step
in treatment?

Figure 1 Figure 2

A. Ultrasonography-guided aspiration
B. Excisional biopsy
C. Observation
D. MRI of the hand for further characterization of the mass
Correct answer: C

 Discussion

The history, clinical photograph, and ultrasonography evaluation point to a diagnosis of ganglion cyst.
The likelihood of resolution of ganglion cysts without intervention in pediatric patients ranges from 66-
79%. Therefore the most appropriate treatment at this time is observation. Because the diagnosis is
confirmed with the information provided, excisional biopsy and further imaging are not necessary.

 Recommended Readings

Coffey MJ, Rahman MF, Thirkannad SM. Pediatric ganglion cysts of the hand and wrist: an
epidemiologic analysis. Hand (N Y). 2008 Dec;3(4):359-62. doi: 10.1007/s11552-008-9122-2. Epub
2008 Jul 16. PubMed

Meyerson J, Pan YL, Spaeth M, Pearson G. Pediatric Ganglion Cysts: A Retrospective Review.
Hand (N Y). 2018 Jan 1:1558944717751195. doi: 10.1177/1558944717751195. [Epub ahead of print]
PubMed

Wang AA, Hutchinson DT. Longitudinal observation of pediatric hand and wrist ganglia. J Hand Surg
Am. 2001 Jul;26(4):599-602. PubMed

Question 6 of 100

What sign or symptom may occur with cubital tunnel syndrome that does not occur with Guyon neuropathy?

A. Abnormal sensation of the dorsal ulnar hand


B. A positive Froment sign
C. Abnormal sensation in the volar ring and small fingers
D. Weakness of the interosseous muscles
 Correct answer: A

Discussion

Ulnar neuropathy at the elbow is termed cubital tunnel syndrome, whereas ulnar nerve compression at
the wrist is considered Guyon neuropathy. Patients with cubital tunnel syndrome have numbness on the
dorsal ulnar aspect of the hand due to involvement of the dorsal ulnar sensory nerve branch (DUSN).
Ulnar neuropathy at both the elbow and the wrist may manifest with abnormal sensation about the volar
ring and small fingers and with weakness of the interosseous muscles, which can lead to a positive
Froment sign. The Froment sign is considered positive when flexion of the thumb interphalangeal joint
occurs to compensate for a lack of adductor function. Patients with a Guyon neuropathy do not have
symptoms of numbness in the dorsal ulnar distribution, because the DUSN branch arises more
proximally in the forearm and is not compressed in the ulnar tunnel at the wrist.

 Recommended Readings

Kenney RJ, Hammert WC. Physical examination of the hand. J Hand Surg Am. 2014 Nov;39(11):2324-
34; quiz 2334. doi: 10.1016/j.jhsa.2014.04.026. Epub 2014 Oct 22. PubMed

Chen SH, Tsai TM. Ulnar tunnel syndrome. J Hand Surg Am. 2014 Mar;39(3):571-9. doi:
10.1016/j.jhsa.2013.08.102. PubMed

Question 7 of 100

Figure 1 shows the radiograph obtained from a 54-year-old woman with rheumatoid arthritis who has thumb
pain and dysfunction. Nonsurgical treatment, including splinting, oral NSAIDs, activity modification, and steroid
injections, has failed. What is the most appropriate surgical intervention?
Figure 1

A. Thumb carpometacarpal (CMC) arthroplasty with ligament suspensionplasty


B. Thumb CMC and thumb metacarpophalangeal (MCP) joint fusion
C. Thumb CMC arthroplasty with ligament suspensionplasty and thumb MCP joint stabilization
D. Trapezial resection and distraction arthroplasty.
Correct answer: C

Discussion

Various options exist to treat thumb CMC arthritis: trapezial resection alone, trapezial resection with
ligament suspensionplasty or tendon interposition, trapezial resection with both ligament
suspensionplasty and tendon interposition, CMC fusion, and CMC replacement.

MCP hyperextension can develop in long-standing CMC arthritis, contributing to CMC instability as
well as thumb pain and weakness. In patients with concomitant MCP hyperextension that exceeds 30°,
correction of the deformity of the MCP joint must also be addressed and can be done with MCP
capsulodesis, extensor pollicis brevis tendon transfer, or MCP fusion. Fusion of both the thumb CMC
and MP joints is not recommended as this would result in marked stiffness and dysfunction.

 Recommended Readings

Qadir R, Duncan SF, Smith AA, Merritt MV, Ivy CC, Iba K. Volar capsulodesis of the thumb
metacarpophalangeal joint at the time of basal joint arthroplasty: a surgical technique using suture
anchors. J Hand Surg Am. 2014 Oct;39(10):1999-2004. doi: 10.1016/j.jhsa.2014.07.045. PubMed

Raskolnikov D, White NJ, Swart E, Zouzias IC, Rosenwasser MP. Volar plate capsulodesis for
metacarpophalangeal hyperextension with basal joint arthritis. Am J Orthop (Belle Mead NJ). 2014
Aug;43(8):354-8. PubMed

Question 8 of 100

Video 1 demonstrates a unilateral "piano key" sign, which indicates


Video 1

A. distal radioulnar joint (DRUJ) instability.


B. interosseous membrane disruption.
C. midcarpal instability.
D. physiologic motion of hypermobility syndrome.

Correct answer: A

Discussion

The piano key sign is a demonstration of instability at the DRUJ, typically seen after healing from a
distal radius fracture with an associated ulnar styloid fracture (as in this case) or other wrist injury. The
hand is pushed down against a table top, and the distal radius translates dorsally (with the distal ulna
apparently moving volarly). In fact, the distal radius is the mobile segment, while the distal ulna is fixed
in space. Treatment involves repair or reconstruction of the foveal insertion of the triangular
fibrocartilage complex (TFCC) and distal radioulnar ligaments.

This type of instability is also common in malunions of the distal radius or distal one-third of the radial
shaft (Galeazzi-type fractures). In malunions, DRUJ instability can be treated with a corrective
osteotomy of the distal radius to restore the anatomic relationship between the distal ulna and the distal
radius at the DRUJ.

Radiocarpal and midcarpal instability do not involve the DRUJ. Disruption of the interosseous
membrane (in isolation, with intact distal radioulnar ligaments and an intact TFCC) does not lead to
translational instability of the DRUJ. Although hypermobility syndrome may lead to ligamentous laxity,
it does not lead to unilateral DRUJ instability.

 Recommended Readings

Moritomo H. The distal interosseous membrane: current concepts in wrist anatomy and biomechanics. J
Hand Surg Am. 2012 Jul;37(7):1501-7. doi: 10.1016/j.jhsa.2012.04.037. Review. PubMed

Mulford JS, Axelrod TS. Traumatic injuries of the distal radioulnar joint. Orthop Clin North Am. 2007
Apr;38(2):289-97, vii. Review. PubMed

Question 9 of 100

Figures 1 and 2 are the radiographs of a 36-year-old man who has had left wrist pain for the past 6 months
following a fall onto his outstretched arm. Examination reveals a positive ballottement test, dorsal and ulnar
carpal tenderness, and a painful snap with ulnar deviation, pronation, and axial compression. Injury to what
ligament is the cause of this patient's pain?
Figure 1 Figure 2

A. Short radiolunate
B. Dorsal scapholunate interosseous
C. Volar lunotriquetral interosseous
D. Radioscaphocapitate

Correct answer: C

Discussion
The radiographs reveal a volarly tilted lunate on the lateral view (Figure 1) and an incongruous
lunotriquetral articulation on the AP gripped view (Figure 2). The patient has what appears to be
radiographic findings of volar intercalated segmental instability (VISI), a type of carpal instability,
dissociative. An injury to the volar lunotriquetral ligament is the most important contributor to this type
of instability. An injury to the dorsal scapholunate ligament typically leads to a dorsal intercalated
segmental instability. Isolated injuries to the dorsal radiocarpal ligament and the radioscaphocapitate
ligament do not lead to VISI, although combined injuries may lead to instability between the radius and
the proximal row.

 Recommended Readings

Atkinson CT, Watson J. Lunotriquetral ligament tears. J Hand Surg Am. 2012 Oct;37(10):2142-4. doi:
10.1016/j.jhsa.2012.04.007. Epub 2012 May 26. PubMed

van de Grift TC, Ritt MJ. Management of lunotriquetral instability: a review of the literature. J Hand
Surg Eur Vol. 2016 Jan; 41 (1): 72-85. doi: 10.1177 / 1753193415595167. Epub 2015 Jul 18. PubMed

Question 10 of 100

A 25-year-old man has an isolated flexor digitorum profundus laceration just proximal to the distal
interphalangeal (DIP) flexion crease of his ring finger. The tendon ends are trimmed, removing 10 mm from each
end (secondary to fraying) and the tendon repaired. Four months later, he reports limited finger motion of the
long, ring, and small fingers. He cannot fully extend his wrist and all joints of the 3 fingers simultaneously. He has
full passive flexion but cannot actively completely close his fingers into a fist. What is the most likely cause?

A. Quadrigia
B. Intrinsic tightness
C. Lumbrical plus deformity
D. Disruption of the tendon repair

Correct answer: A
Discussion

If a single flexor digitorum profundus (FDP) tendon is debrided more than 1 cm prior to repair, the
tendon is advanced too far distally, essentially shortening the musculotendon unit. The finger will likely
develop a flexion posture. Because of the common muscle belly and interconnections of the profundi,
the long and small fingers adjacent to the injured finger will be affected because of loss of some of their
normal proximal excursion. The result is an inability of the adjacent fingers to completely flex. This
condition, known as quadrigia, is named after the Roman chariot driver who held control of the reins of
4 horses, forcing them to move as 1. Quadrigia occurs when the FDP tendon is advanced too far distally,
when a tendon graft is too short, or when the profundus is sutured over the end of an amputated digit.

Intrinsic muscles of the hand flex the metacarpophalangeal (MP) joints and extend the PIP joint.
Intrinsic tightness causes decreased PIP flexion when the MP joint is in extension. The lumbrical muscle
modulates tension on the flexor profundus tendon. When a tendon graft to repair the profundus tendon is
too long, a lumbrical plus deformity occurs. This is a paradoxical PIP extension as the finger is flexed.
Disruption of the tendon repair causes limited flexion of the injured finger.

 Recommended Readings

Schreuders TA. The quadriga phenomenon: a review and clinical relevance. J Hand Surg Eur Vol. 2012
Jul;37(6):513-22. doi: 10.1177/1753193411430810. Epub 2011 Dec 14. Review. PubMed

Malerich MM, Baird RA, McMaster W, Erickson JM. Permissible limits of flexor digitorum profundus
tendon advancement--an anatomic study. J Hand Surg Am. 1987 Jan; 12(1):30-3. PubMed

Question 11 of 100

Figure 1 shows the radiograph obtained from a 67-year-old woman who has progressive wrist pain. She
undergoes a salvage motion-sparing surgery that relies on the intact cartilage of the capitate head. It is
necessary to preserve what structure during this procedure?
Figure 1

A. Long radiolunate ligament


B. Radioscaphocapitate ligament
C. Dorsal radiocarpal ligament
D. Dorsal intercarpal ligament

Correct answer: B

 Discussion

The radioscaphocapitate ligament must be preserved in cases of proximal row carpectomy


or scaphoidectomy with four-corner fusion. Failure to do so can result in ulnar translocation of the
carpus. The attachment of the long radiolunate ligament to the lunate is compromised in proximal row
carpectomy, although it is left intact in scaphoidectomy with four-corner fusion. Preservation of the
dorsal radiocarpal ligament through a limited arthrotomy is advocated by Ozyurekoglu and Turker as a
method of preserving the blood and nerve supply of the carpus, but this technique has not been proven to
be required. The authors did cut through the dorsal intercarpal ligament in their exposure.

 Recommended Readings

Green DP, Perreira AC, Longhofer LK. Proximal Row Carpectomy. J Hand Surg Am. 2015
Aug;40(8):1672-6. doi: 10.1016/j.jhsa.2015.04.033. Epub 2015 Jun 25. PubMed

Ozyurekoglu T, Turker T. Results of a method of 4-corner arthrodesis using headless compression


screws. J Hand Surg Am. 2012 Mar;37(3):486-92. doi: 10.1016/j.jhsa.2011.12.022. Epub 2012 Feb 1.
PubMed

Question 12 of 100

At which joint do degenerative changes occur first in a patient with chronic, untreated scapholunate
dissociation?

A. Radioscaphoid
B. Radiolunate
C. Scapholunate
D. Capitolunate

Correct answer: A

 Discussion

Stage I of scapholunate advanced collapse (SLAC) is characterized by the presence of radioscaphoid


arthritis. A predictable pattern exists of the progression of degenerative changes for SLAC wrist,
including stage I (radial styloid involvement at the scaphoid fossa), stage II (scaphoid and entire
scaphoid facet involvement), stage III (degeneration between the capitate and lunate), and stage IV
(pancarpal involvement). The radiolunate joint is often spared.
 Recommended Readings

Kitay A, Wolfe SW. Scapholunate instability: current concepts in diagnosis and management. J Hand
Surg Am. 2012 Oct;37(10):2175-96. doi: 10.1016/j.jhsa.2012.07.035. PubMed

Watson HK, Weinzweig J, Zeppieri J. The natural progression of scaphoid instability. Hand Clin. 1997
Feb;13(1):39-49. PubMed

Question 13 of 100

Figures 1 through 3 are the radiographs of a 55-year-old woman who underwent a volar plating of an extra-
articular distal radius fracture 2 weeks ago. She is experiencing weakness with flexion of the interphalangeal (IP)
thumb joint. IP joint flexion was normal before surgery. What is the best next step?
Figure 1 Figure 2

Figure 3

A. Observation
B. Electromyogram/nerve conduction study (EMG/NCS)
C. CT scan
D. Immediate exploration
Correct answer: A

 Discussion

Figure 4

Prevalence of flexor tendon rupture after distal radius fracture is between 2% and 12%. The FPL tendon
is the most common flexor tendon rupture associated with volar plating. It is usually seen with plates
that are distal to the watershed line (W) and with plates extending volar to the critical line (C) (Figure 4).
The watershed line (W) is the location of the origin of the volar carpal ligaments and the bone
prominence at which flexor tendons are most closely opposed to the distal radius (Figure 4). In this
scenario, the lateral radiograph shows that the plate is not distal to the watershed line (W) and is
between the critical line (C) and the line parallel to the volar cortex of the radius (R). This is the optimal
position for the plate. Placement of a volar locking plate distal to the watershed line of the distal radius
and excessive plate prominence has been associated with FPL tendon rupture.

This patient is only 2 weeks past surgery and there is some FPL function. FPL weakness after volar
distal radius plating is common and has been seen in as many as 50% of patients. This usually recovers
spontaneously by 2 months, and no treatment is needed. A nerve conduction study would be indicated if
an anterior interosseous nerve compression were considered, but it is too early for this test. A CT scan
could be obtained to judge the alignment of the fracture fragment and position of the screws, but it is not
indicated in this case. Exploration could be performed if an FPL rupture were considered, but, because it
is only 2 weeks after surgery, there is some FPL function, the plate is proximal to the watershed line,
and immediate exploration is not indicated. If this does not improve after 2 to 3 months, further
investigation with ultrasound or MRI would be indicated.

 Recommended Readings

Griffin JW, Chhabra AB. Complications after volar plating of distal radius fractures. J Hand Surg Am.
2014 Jun;39(6):1183-5; quiz 1186. doi: 10.1016/j.jhsa.2014.03.038. Epub 2014 May 5. Review.
PubMed

Agnew SP, Ljungquist KL, Huang JI. Danger zones for flexor tendons in volar plating of distal radius
fractures. J Hand Surg Am. 2015 Jun;40(6):1102-5. doi: 10.1016/j.jhsa.2015.02.026. Epub 2015 Apr 2.
PubMed

Soong M, Earp BE, Bishop G, Leung A, Blazar P. Volar locking plate implant prominence and flexor
tendon rupture. J Bone Joint Surg Am. 2011 Feb 16;93(4):328-35. doi: 10.2106/JBJS.J.00193. Epub
2011 Jan 14. PubMed

Chilelli BJ, Patel RM, Kalainov DM, Peng J, Zhang LQ. Flexor pollicis longus dysfunction after volar
plate fixation of distal radius fractures. J Hand Surg Am. 2013 Sep;38(9):1691-7. doi:
10.1016/j.jhsa.2013.06.005. Epub 2013 Jul 30. PubMed

Question 14 of 100

The decision to perform fasciotomy of the fingers for a hand compartment syndrome is most appropriately
made using

A. clinical examination.
B. invasive pressure measurement.
C. arterial Doppler study.
D. MRI.
Correct answer: A

 Discussion

Compartment syndrome of the hand can result from a variety of factors, including a traumatic event
such as crush injury, fracture, vascular insult, a high-pressure injection injury, or an insect or spider bite.
The treatment involves decompressive fasciotomy of the involved compartments. The diagnosis of hand
compartment syndrome is determined by history, examination, and objective testing. Patients experience
pain out of proportion to the injury, along with swelling and tense skin. Pain may occur with passive
motion of the metacarpophalangeal joints as the intrinsic muscles are stretched. Invasive
intracompartmental pressures can be measured in the compartments of the hand but not in the fingers.
Arterial Doppler studies assess arterial blood flow, and an abnormality would be a late finding. MRI
would show edema of the hand and fingers, but the decision to perform surgical release is less likely
made from the findings. The most appropriate method of determining the need for finger fasciotomy is
the history and physical examination.

 Recommended Readings

Codding JL, Vosbikian MM, Ilyas AM. Acute compartment syndrome of the hand. J Hand Surg Am.
2015 Jun;40(6):1213-6; quiz 1216. doi: 10.1016/j.jhsa.2015.01.034. Epub 2015 Mar 20. PubMed

Lipschitz AH, Lifchez SD. Measurement of compartment pressures in the hand and forearm. J Hand
Surg Am. 2010 Nov;35(11):1893-4. doi: 10.1016/j.jhsa.2010.07.006. Epub 2010 Sep 25. PubMed

Question 15 of 100

Figure 1 is the radiograph of an 18-year-old, right hand-dominant man who has right side thumb pain after a
tackle during a rugby game. Examination shows ecchymosis and swelling of the right thumb along with
tenderness to palpation about the thumb CMC joint and metacarpal base. What ligament is holding the small
fracture fragment in anatomical location to the trapezium?
Figure 1

A. Posterior oblique
B. Dorsal radial
C. Anterior oblique
D. Dorsal trapeziometacarpal

Correct answer: C

 Discussion

Bennett fractures are defined as intra-articular thumb metacarpal base fractures. The fracture is often
caused by axial loading, and concomitant injuries to the thumb MCP joint and trapezium are common.
The palmar ulnar aspect of the base of the metacarpal stays in place through its attachment to the
trapezium by way of the anterior oblique ligament. The metacarpal shaft is displaced dorsally,
proximally, and radially by the pull of the abductor pollicis longus, extensor pollicis brevis, extensor
pollicis longus, and adductor pollicis brevis. These fractures are often considered unstable and are
treated surgically.

 Recommended Readings

Pavić R, Malović M. Operative treatment of Bennett's fracture. Coll Antropol. 2013 Mar;37(1):169-74.
PubMed

Carter KR, Nallamothu SV. Fracture, Bennett. 2018 May 13. StatPearls [Internet]. Treasure Island (FL):
StatPearls Publishing; 2018 Jan. Available from http://www.ncbi.nlm.nih.gov/books/NBK500035/ Last
accessed 12-13-2018.

Rivlin M, Fei W, Mudgal CS. Bennett Fracture. J Hand Surg Am. 2015 Aug;40(8):1667-8. doi:
10.1016/j.jhsa.2015.05.017. Epub 2015 Jul 3. PubMed

Question 16 of 100

A 45-year-old woman has a distal radius fracture, which is treated with open reduction and internal fixation. The
surgery was uncomplicated, and the patient is discharged to home. At the first follow-up appointment, the
patient demonstrates signs that are concerning for complex regional pain syndrome (CRPS). What factor is
included in the International Association for the Study of Pain (IASP) criteria (Budapest criteria) for the diagnosis
of CRPS?

A. Hypoesthesia
B. Elevated white blood cell count
C. Elevated C-reactive protein level
D. Pain disproportionate to the inciting event

Correct answer: D

 Discussion
The diagnosis of CRPS is complex. The IASP has approved diagnostic criteria to standardize both the
diagnosis and the study of CRPS. The criteria are:

1. Continuing pain disproportionate to any inciting event


2. At least one symptom in three of the following four categories
1. Sensory: reports of hyperesthesia and/or allodynia
2. Vasomotor: reports of temperature asymmetry and/or skin color changes and/or skin
color asymmetry
3. Sudomotor/edema: reports of edema and/or sweating changes and/or sweating asymmetry
4. Motor/trophic: reports of decreased range of motion (ROM) and/or motor dysfunction
(weakness, tremor, dystonia) and/or trophic changes (hair, skin, nails)
3. At least one sign at the time of evaluation in two (for clinical diagnosis) or three (for inclusion in
scientific studies) of the following categories
1. Sensory: evidence of hyperesthesia (to pinprick) and/or allodynia (to light touch and/or
deep somatic pressure and/or joint movement)
2. Vasomotor: evidence of temperature asymmetry and/or skin color changes and/or skin
color asymmetry
3. Sudomotor/edema: evidence of edema and/or sweating changes and/or sweating
asymmetry
4. Motor/trophic: evidence of decreased ROM and/or motor dysfunction (weakness, tremor,
dystonia) and/or trophic changes (hair, skin, nails)
4. No other diagnosis better explains the signs and symptoms.

Decreased sensation and focal numbness are not consistent with CRPS. Laboratory and imaging studies
can be helpful in evaluating for the exclusion of differential diagnoses for CRPS, including infection,
rheumatic disease, fracture, nonunion, tenosynovitis, or osteomyelitis.

 Recommended Readings

Koh TT, Daly A, Howard W, Tan C, Hardidge A. Complex Regional Pain Syndrome. JBJS Rev. 2014
Jul 22;2(7). pii: 01874474-201402070-00004. doi: 10.2106/JBJS.RVW.M.00085. PubMed

Birklein F, Schlereth T. Complex regional pain syndrome-significant progress in understanding. Pain.


2015 Apr;156 Suppl 1:S94-103. doi: 10.1097/01.j.pain.0000460344.54470.20. PubMed

Question 17 of 100
According to clinical and biomechanical studies, the most appropriate position for a headless scaphoid
compression screw for repair of a scaphoid waist fracture is

A. retrograde to protect the dorsal blood supply to the scaphoid.


B. retrograde eccentrically in the dorsal scaphoid to avoidtrapezium impingement.
C. deep and centrally placed, respecting the articular surface.
D. anterograde to protect the volar blood supply to the scaphoid.

Correct answer: C

 Discussion

Figure 1
The position of a scaphoid screw for scaphoid fracture repair (Figure 1) is as critical as the position of a
sliding hip screw for intertrochanteric fracture repair. Positioning the screw deep in the center of the
densest portion of cancellous bone is beneficial for both of these fracture types.

Trumble and associates have shown time to union for scaphoid nonunions to be decreased for centrally
placed scaphoid screws. McCallister and associates documented improved biomechanical stability for
scaphoid waist fractures repaired with a centrally placed screw vs an eccentrically placed screw. Dodds
and associates demonstrated significantly improved biomechanical stability with centrally placed long
screws vs centrally placed short screws.

The screw may be placed retrograde or antegrade. Although the screw position may be relatively parallel
to the radial inclination as the shape of the scaphoid body follows the contour of the radial styloid, screw
position should be assessed relative to the scaphoid’s own architecture. Placing the screw in a retrograde
fashion can force the treating surgeon to start the screw eccentrically in an attempt to avoid the
interference of the trapezium lying over the distal pole of the scaphoid. This may result in noncentral
screw placement.

 Recommended Readings

Dodds SD, Panjabi MM, Slade JF 3rd. Screw fixation of scaphoid fractures: a biomechanical assessment
of screw length and screw augmentation. J Hand Surg Am. 2006 Mar;31(3):405-13. PubMed

McCallister WV, Knight J, Kaliappan R, Trumble TE. Central placement of the screw in simulated
fractures of the scaphoid waist: a biomechanical study. J Bone Joint Surg Am. 2003 Jan;85-A(1):72-7.
PubMed

Trumble TE, Clarke T, Kreder HJ. Non-union of the scaphoid. Treatment with cannulated screws
compared with treatment with Herbert screws. J Bone Joint Surg Am. 1996 Dec;78(12):1829-37.
PubMed

Question 18 of 100

Figures 1 and 2 show the intraoperative photographs obtained during surgical treatment for de Quervain
tendonitis. For orientation purposes, dorsal is at the top. Figure 1 is obtained just after the initial first extensor
compartment release, and Figure 2 shows the floor of the first extensor compartment. If the structure marked
by the black dot is not addressed, the most common postoperative problem would be
Figure 1 Figure 2

A. persistent pain.
B. tendon subluxation.
C. altered sensation.
D. tendon rupture.

Correct answer: A

 Discussion

The black dot identifies an accessory compartment of the extensor pollicis brevis (EPB) tendon. The
incidence of accessory EPB compartment in patients undergoing surgical treatment for de Quervain
syndrome ranges from 46% to 60%. Failure to release this compartment at the time of initial surgery can
cause persistent postoperative pain.

The patient would not experience altered sensation if this compartment were not released. Altered
sensation would most commonly occur following injury to the dorsal radial sensory nerve branch during
surgery. EPB tendon subluxation also would not occur should the accessory compartment not be
released. For EPB tendon subluxation to occur, its own compartment would need to be released first.
Finally, EPB tendon rupture would be an extremely uncommon complication of failure to release the
accessory compartment.
 Recommended Readings

Weiss AP, Akelman E, Tabatabai M. Treatment of de Quervain's disease. J Hand Surg Am. 1994
Jul;19(4):595-8. PubMed

Ilyas AM, Ast M, Schaffer AA, Thoder J. De quervain tenosynovitis of the wrist. J Am Acad Orthop
Surg. 2007 Dec;15(12):757-64. Review. Erratum in: J Am Acad Orthop Surg. 2008 Feb;16(2):35A.
Ilyas, Asif [corrected to Ilyas, Asif M]. Full text

Question 19 of 100

Video 1 depicts a 20-year-old right-hand-dominant man with a 6-month history of left wrist pain and popping
that has failed nonsurgical measures. No other positive findings upon examination are noted. What is the most
appropriate course of treatment?

Video 1

A. Triangular fibrocartilage complex (TFCC) repair


B. Lunotriquetral fusion
C. Distal radioulnar joint (DRUJ) tenodesis
D. Extensor carpi ulnaris (ECU) tendon sheath reconstruction

Correct answer: D
 Discussion

Upon examination, this patient is exhibiting dislocation of the ECU tendon because of a disrupted
sheath. He has failed nonsurgical measures, so surgery that would involve either direct repair or
reconstruction of the tendon sheath is indicated. An option for reconstruction is to use a portion of the
extensor retinaculum as a sheath substitute. Deepening of the ECU tendon groove at the distal ulna with
direct repair of the sheath is another option, although a 2016 paper by Ghatan and associates did not find
depth of the groove as a risk factor for subluxation. TFCC repair, lunotriquetral fusion, and DRUJ
tenodesis are not appropriate because the examination clearly shows ECU tendon dislocation. TFCC and
lunotriquetral ligament tears can occur along with ECU tendon dislocation, but no other examination
findings suggest these conditions for this patient.

 Recommended Readings

Ghatan AC, Puri SG, Morse KW, Hearns KA, von Althann C, Carlson MG. Relative Contribution of the
Subsheath to Extensor Carpi Ulnaris Tendon Stability: Implications for Surgical Reconstruction and
Rehabilitation. J Hand Surg Am. 2016 Feb;41(2):225-32. doi: 10.1016/j.jhsa.2015.10.024. Epub 2015
Dec 12. PubMed

Inoue G, Tamura Y. Surgical treatment for recurrent dislocation of the extensor carpi ulnaris tendon. J
Hand Surg Br. 2001 Dec;26(6):556-9. PubMed

Allende C, Le Viet D. Extensor carpi ulnaris problems at the wrist--classification, surgical treatment and
results. J Hand Surg Br. 2005 Jun;30(3):265-72. Epub 2005 Apr 7. PubMed

MacLennan AJ, Nemechek NM, Waitayawinyu T, Trumble TE. Diagnosis and anatomic reconstruction
of extensor carpi ulnaris subluxation. J Hand Surg Am. 2008 Jan;33(1):59-64. doi:
10.1016/j.jhsa.2007.10.002. PubMed

Question 20 of 100

Figures 1 and 2 are of a 51-year-old man who underwent open reduction and internal fixation of a right proximal
humerus fracture with concomitant rotator cuff repair. Within 1 year, he develops heterotopic ossification, for
which he undergoes excision and hardware removal. Postoperatively, he was noted to have progressive atrophy
in the shoulder and anterior humeral head subluxation with attempted shoulder abduction. What nerve was
damaged during the most recent procedure?
Figure 1 Figure 2

A. Suprascapular
B. Radial
C. Anterior branch of axillary
D. Spinal accessory (cranial nerve XI)

Correct answer: C

 Discussion
This patient has a deficiency of the anterior deltoid muscle, resulting in inferior subluxation of the
humerus with associated glenohumeral instability. Axillary nerve injury during shoulder surgery
accounts for 6% to 10% of brachial plexus injuries. In the posterior scapular region, the axillary nerve
terminates by dividing into two main branches: the posterior terminal branch, which provides motor
innervation to the teres minor and posterior deltoid muscles, and the anterior terminal branch, which
provides motor innervation to the anterior and middle portions of the deltoid muscle. The deltoid
determines the silhouette of the shoulder and is a stabilizer of the humeral head.

 Recommended Readings

Moser T, Lecours J, Michaud J, Bureau NJ, Guillin R, Cardinal É. The deltoid, a forgotten muscle of the
shoulder. Skeletal Radiol. 2013 Oct;42(10):1361-75. doi: 10.1007/s00256-013-1667-7. Epub 2013 Jun
20. Review. PubMed.

Stecco C, Gagliano G, Lancerotto L, Tiengo C, Macchi V, Porzionato A, De Caro R, Aldegheri R.


Surgical anatomy of the axillary nerve and its implication in the transdeltoid approaches to the shoulder.
J Shoulder Elbow Surg. 2010 Dec;19(8):1166-74. doi: 10.1016/j.jse.2010.05.010. Epub 2010 Aug 25.
PubMed

Question 21 of 100

A 65-year-old right–hand-dominant woman has been experiencing thenar and wrist pain for 18 months. She has
no history of trauma. The pain worsens during the opening of jars, grasping, writing, and repetitive thumb use.
Examination reveals tenderness to palpation over the volar thenar eminence, just distal to the scaphoid
tubercle, and along the flexor carpi radialis sheath. A Watson scaphoid shift test produces pain but no instability
or clunk. Radiographs reveal isolated scaphotrapeziotrapezoidal (STT) arthritis with mild dorsal intercalated
segment instability (DISI) deformity. She has worn a splint on and off for the past year, has had multiple
cortisone injections, and has modified her activity, all of which helped initially. She wants to move forward with
surgical intervention. STT arthrodesis is chosen over distal pole scaphoid excision. What factor in her evaluation
indicates that arthrodesis would be preferred over distal pole excision?

A. Failure of pain relief from steroid injection and NSAID use


B. Tenderness that is distal to the scaphoid tubercle
C. Isolated STT arthritis on radiograph
D. Mild DISI deformity on radiograph
Correct answer: D

 Discussion

Isolated STT arthritis is common and can cause substantial patient disability and pain. After nonsurgical
treatment has failed, surgical intervention is warranted. Surgical options include distal pole scaphoid
excision, STT arthrodesis, or carpometacarpal (CMC) arthroplasty, if concomitant thumb CMC
degenerative changes are present. Distal pole scaphoid excision is less commonly employed for the
treatment of STT arthritis, because of the potential development of intercalated segmental instability.
However, distal pole scaphoid excision is less technically demanding, engenders fewer surgical
complications, and promotes a faster return to previous activity levels. In any patient with preoperative
carpal malalignment, removing the distal pole of the scaphoid would exacerbate that deformity and
could lead to symptoms from the DISI deformity. Thus, in this patient with mild DISI deformity seen on
preoperative radiographs, STT arthrodesis is the most appropriate treatment option.

 Recommended Readings

Zimmermann MS, Weiss AP. Scaphotrapezium-trapezoid arthrosis. J Hand Surg Am. 2012
Oct;37(10):2139-41; quiz 2141. doi: 10.1016/j.jhsa.2012.05.007. Epub 2012 Jul 3. PubMed

Garcia-Elias M. Excisional arthroplasty for scaphotrapeziotrapezoidal osteoarthritis. J Hand Surg Am.


2011 Mar;36(3):516-20. doi: 10.1016/j.jhsa.2010.12.016. PubMed

Kapoutsis DV, Dardas A, Day CS. Carpometacarpal and scaphotrapeziotrapezoid arthritis: arthroscopy,
arthroplasty, and arthrodesis. J Hand Surg Am. 2011 Feb;36(2):354-66. doi: 10.1016/j.jhsa.2010.11.047.
PubMed

Tay SC, Moran SL, Shin AY, Linscheid RL. The clinical implications of scaphotrapezium-trapezoidal
arthritis with associated carpal instability. J Hand Surg Am. 2007 Jan;32(1):47-54. PubMed

Question 22 of 100

Nerve conduction velocity is slowed by


A. increased skin temperature.
B. increased perineural blood flow.
C. external compression.
D. hand dominance.

Correct answer: C

 Discussion

A number of factors affect nerve conduction velocity; for example, increased body temperature
increases nerve conduction velocity. Nerve conduction velocity is slowed by advancing age,
compression, decreased blood flow, and fibrosis (from large imprecise sutures used for nerve repair).
There is no association between hand dominance and nerve conduction velocity.

 Recommended Readings

Lundborg G. A 25-year perspective of peripheral nerve surgery: evolving neuroscientific concepts and
clinical significance. J Hand Surg Am. 2000 May;25(3):391-414. Review. PubMed

Freedman M, Helber G, Pothast J, Shahwan TG, Simon J, Sher L. Electrodiagnostic evaluation of


compressive nerve injuries of the upper extremities. Orthop Clin North Am. 2012 Oct;43(4):409-16.
doi:10.1016/j.ocl.2012.07.010. Epub 2012 Sep 10. Review. PubMed

Dodds SD. Peripheral Nervous System. In Boyer MI, ed. AAOS Comprehensive Orthopaedic Review.
Vol 1. 2nd ed. Rosemont, IL: American Academy of Orthopaedic Surgeons; 2014:113-126

Question 23 of 100

Figures 1 through 3 demonstrate the MRIs obtained from a 36-year-old man with an injury to the elbow. The
expected result of nonsurgical treatment would be weakness of
Figure 1 Figure 2 Figure 3

A. finger flexion.
B. elbow extension.
C. finger extension.
D. forearm supination.

Correct answer: D

 Discussion

The axial T2-weighted MRIs demonstrate a distal biceps rupture. The increased signal is noted
superficial to the brachialis muscle and adjacent to the biceps tuberosity. The distal biceps tendon is not
seen in the distal cuts and has retracted proximally. The physical examination of patients with these
injuries show abnormal contour of the arm and tenderness in the antecubital fossa. The hook test is a
provocative maneuver that documents biceps integrity. When performing the maneuver, the examiner
attempts to hook a finger around the distal biceps tendon while the patient actively supinates with the
elbow held in flexion. Nonsurgical treatment has been documented to result in an average loss of 40% of
supination strength and 30% of elbow flexion strength. Repair is optimal within several weeks of injury.
The alternative options would not occur with a distal biceps rupture.

 Recommended Readings
Sutton KM, Dodds SD, Ahmad CS, Sethi PM. Surgical treatment of distal biceps rupture. J Am Acad
Orthop Surg. 2010 Mar;18(3):139-48. Full text

Baratz M, King GJ, Steinmann S. Repair of distal biceps ruptures. J Hand Surg Am. 2012
Jul;37(7):1462-6. doi: 10.1016/j.jhsa.2012.02.008. Epub 2012 Apr 4. PubMed

Question 24 of 100

A 44-year-old man sustains the injury shown in Figures 1 through 3. What is the most appropriate treatment?

Figure 1 Figure 2 Figure 3

A. Reduction and internal fixation


B. Closed reduction and splinting alone
C. Carpometacarpal arthrodesis
D. Carpometacarpal (CMC) joint suspension arthroplasty
Correct answer: A

 Discussion

Reduction, either open or closed, with internal fixation (pinning) is the recommended treatment for the
majority of these injuries. Closed reduction with pinning is most often performed for acute injuries.
Open reduction with pinning is performed for those injuries that cannot be reduced by closed means or
those with a delayed presentation.

Four cases of successful closed reduction and splinting, all performed upon presentation in the
emergency department, have been described by Storken and associates, but the authors note that their
review of three prior reports uncovered cases of secondary dislocation, which required surgical
stabilization. One of the dislocations occurred 4 months after the reduction. They assert that an
indication for primary ORIF is a CMC dislocation associated with major fractures.

Primary arthrodesis can be considered in cases with severe intra-articular comminution, but this
procedure substantially limits the ability of the hand to increase and decrease the transverse metacarpal
arch, which is an important functional movement. It can also lead to osteoarthritis of the triquetrohamate
joint.

Suspension arthroplasty has been described for old fracture-dislocations of the fifth CMC joint, using a
partial slip of the extensor carpi ulnaris.
 Recommended Readings

Smith GR, Yang SS, Weiland AJ. Multiple carpometacarpal dislocations. A case report and review of
treatment. Am J Orthop (Belle Mead NJ). 1996 Jul;25(7):502-6. PubMed

Storken G, Bogie R, Jansen EJ. Acute ulnar carpometacarpal dislocations. Can it be treated
conservatively? A review of four cases. Hand (N Y). 2011 Dec;6(4):420-3. doi: 10.1007/s11552-011-
9347-3. Epub 2011 Jul 7. PubMed

Kato N, Fukumoto K. Suspension Arthroplasty for Old Fracture-Dislocations of the Fifth


Carpometacarpal Joint. Tech Hand Up Extrem Surg. 2016 Jun;20(2):83-7. doi:
10.1097/BTH.0000000000000120. PubMed
Question 25 of 100

What is the most common complication after distal biceps tendon repair at the elbow?

A. Lateral antebrachial cutaneous neuritis


B. Radial sensory neuritis
C. Symptomatic heterotopic ossification
D. Rupture of the repair

Correct answer: A

 Discussion

Cain and associates retrospectively reviewed 198 consecutive surgical repairs of the biceps and noted a
36% overall complication rate, including 26% paresthesia of the lateral antebrachial cutaneous nerve,
6% paresthesia of the sensory branch of the radial nerve, 2% superficial infection, 4% injury to the
posterior interosseous nerve, 3% symptomatic heterotopic ossification, and 2% rerupture.

Grewal and associates randomized 47 patients to have single-incision repair with two suture anchors and
44 patients to have two-incision transosseous suture repair. They reported that the single-incision
technique had a higher rate of transient neurapraxia to the lateral antebrachial cutaneous nerve, affecting
19 of 47 in the suture anchor group and three of 43 in the transosseous group. Also, four reruptures
occurred, which appeared to be independent of the repair technique.

Recordon and associates found only three complications in their series of 46 patients, 19 having
endobutton repair and 27 with transosseous suture repair. They reported two injuries to the lateral
antebrachial cutaneous nerve and one case of heterotopic ossification.

The review by Banerjee and associates showed one rupture of repair (disengagement of the cortical
button), two persistent lesions of the superficial branch of the radial nerve, and one symptomatic
heterotopic ossification.

Cohen remarked that rerupture of the tendon after repair is uncommon in both one-incision and two-
incision techniques.
Van den Bogaerde and Shin presented a case of posterior interosseous nerve incarceration with
endobutton repair.

 Recommended Readings

Grewal R, Athwal GS, MacDermid JC, Faber KJ, Drosdowech DS, El-Hawary R, King GJ. Single
versus double-incision technique for the repair of acute distal biceps tendon ruptures: a randomized
clinical trial. J Bone Joint Surg Am. 2012 Jul 3;94(13):1166-74. doi: 10.2106/JBJS.K.00436. PubMed

Banerjee M, Shafizadeh S, Bouillon B, Tjardes T, Wafaisade A, Balke M. High complication rate


following distal biceps refixation with cortical button. Arch Orthop Trauma Surg. 2013
Oct;133(10):1361-6. doi: 10.1007/s00402-013-1819-1. Epub 2013 Jul 24. PubMed

Recordon JA, Misur PN, Isaksson F, Poon PC. Endobutton versus transosseous suture repair of distal
biceps rupture using the two-incision technique: a comparison series. J Shoulder Elbow Surg. 2015 Jun;
24 (6): 928-33. doi: 10.1016 / j.jse.2014.12.032. Epub 2015 Apr 7. PubMed

Van den Bogaerde J, Shin E. Posterior interosseous nerve incarceration with endobutton repair of distal
biceps. Orthopedics. 2015 Jan;38(1):e68-71. doi: 10.3928/01477447-20150105-92. PubMed

Cohen MS. Complications of distal biceps tendon repairs. Sports Med Arthrosc Rev. 2008
Sep;16(3):148-53. doi: 10.1097/JSA.0b013e3181824eb0. PubMed

Cain RA, Nydick JA, Stein MI, Williams BD, Polikandriotis JA, Hess AV. Complications following
distal biceps repair. J Hand Surg Am. 2012 Oct;37(10):2112-7. doi: 10.1016/j.jhsa.2012.06.022. Epub
2012 Aug 30. PubMed

Question 26 of 100

A 67-year-old woman has a painful, arthritic proximal interphalangeal (PIP) joint, and nonsurgical measures have
failed to improve the pain. What implant and joint replacement approach combination has been demonstrated
to have the lowest rate of revision surgery?

A. Silicone replacement arthroplasty through a volar approach


B. Surface replacement arthroplasty through a volar approach
C. Silicone replacement arthroplasty through a dorsal approach
D. Surface replacement arthroplasty through a dorsal approach

Correct answer: A

 Discussion

A recent systematic review compared silicone replacement, pyrocarbon replacement, and surface
replacement arthroplasty for PIP arthritis. Silicone arthroplasty through a volar approach showed the
greatest gains in arc of motion and had the lowest rate of revision surgeries. The rates of revision
surgeries from low to high for each type of arthroplasty were 6% for silicone volar, 10% for silicone
lateral, 11% for silicone dorsal, 18% for surface replacement dorsal, and 37% for surface replacement
volar. Revision surgeries include implant replacement (to silicone or maintaining the surface
replacement), arthrodesis, explantation, amputation, and other procedures. Surface replacement
arthroplasty through a volar approach showed the highest revision rate, the worst gain in arc of motion,
and the greatest extension lag. However, substantial pain relief and higher satisfaction still were reported
after surface replacement arthroplasty, regardless of the complications.

 Recommended Readings

Yamamoto M, Chung KC. Implant Arthroplasty: Selection of Exposure and Implant. Hand Clin. 2018
May;34(2):195-205. doi: 10.1016/j.hcl.2017.12.009. PubMed

Yamamoto M, Malay S, Fujihara Y, Zhong L, Chung KC. A Systematic Review of Different Implants
and Approaches for Proximal Interphalangeal Joint Arthroplasty. Plast Reconstr Surg. 2017
May;139(5):1139e-1151e. doi: 10.1097/PRS.0000000000003260. PubMed

Question 27 of 100

Figures 1 through 4 are the radiographs and MR images of a healthy 21-year-old woman who has had persistent
dorsal wrist pain despite immobilization and no history of trauma. The surgical procedure associated with the
best prognosis in this scenario is
Figure 1 Figure 2
Figure 3 Figure 4

A. capitate excision with interposition arthroplasty.


B. capitate proximal pole excision and drilling.
C. proximal row carpectomy (PRC).
D. vascularized bone graft.

Correct answer: D

 Discussion

This patient has osteonecrosis of the capitate. The MR images show evidence of osteonecrosis with
decreased signal on the T1-weighted image. The radiographs are unremarkable, with the exception of
lunotriquetral coalition, which does not necessitate treatment. The etiology of osteonecrosis of the
capitate may be related to trauma, abnormal Interosseous vascular supply, and hypermobility. Surgery is
an option for patients with persistent symptoms despite immobilization. Vascularized bone graft should
be considered in this scenario because there is no evidence of capitate collapse or arthritic change about
the wrist. Free and local vascularized bone grafts have produced satisfactory results. Capitate excision
with interposition arthroplasty is indicated for patients with proximal pole capitate collapse. Total wrist
fusion is a salvage procedure and would be considered if there were evidence of collapse and arthritic
change. PRC would leave the capitate articulating with the radius and is not indicated.
 Recommended Readings

Peters SJ, Degreef I, De Smet L. Avascular necrosis of the capitate: report of six cases and review of the
literature. J Hand Surg Eur Vol. 2015 Jun;40(5):520-5. doi: 10.1177/1753193414524876. Epub 2014
Feb 25. Review. PubMed

Lapinsky AS, Mack GR. Avascular necrosis of the capitate: a case report. J Hand Surg Am. 1992
Nov;17(6):1090-2. Review. PubMed

Hattori Y, Doi K, Sakamoto S, Yukata K, Shafi M, Akhundov K. Vascularized pedicled bone graft for
avascular necrosis of the capitate: case report. J Hand Surg Am. 2009 Sep;34(7):1303-7. doi:
10.1016/j.jhsa.2009.04.012. Epub 2009 Jun 4. PubMed

Question 28 of 100

What vitamin supplement has been shown in some studies to reduce the risk of complex regional pain
syndrome following a distal radius fracture?

A. A
B. B
C. C
D. D

Correct answer: C

 Discussion

Two studies have shown that supplemental vitamin C reduces the risk of developing complex regional
pain syndrome following a distal radius fracture. The recommended dose is 500 mg daily for 50 days.
Supplemental vitamin C is a recommendation of the AAOS evidence-based Clinical Practice Guidelines
and has moderate evidence. The vitamin supplements listed as alternative options have not been shown
to prevent disproportionate pain following a distal radius fracture.
 Recommended Readings

American Academy of Orthopaedic Surgeons: The Treatment of Distal Radius Fractures. Rosemont, IL:
American Academy of Orthopaedic Surgeons, December 2009. Available at www.orthoguidelines.org
Accessed 06-June-2018.

Zollinger PE, Tuinebreijer WE, Breederveld RS, Kreis RW. Can vitamin C prevent complex regional
pain syndrome in patients with wrist fractures? A randomized, controlled, multicenter dose-response
study. J Bone Joint Surg Am. 2007 Jul;89(7):1424-31. PubMed

Koval K, Haidukewych GJ, Service B, Zirgibel BJ. Controversies in the management of distal radius
fractures. J Am Acad Orthop Surg. 2014 Sep;22(9):566-75. doi: 10.5435/JAAOS-22-09-566. Full text

Question 29 of 100

Figures 1 through 3 demonstrate the radiographs obtained from a 25-year-old man who injured his right, by
punching a wall 3 weeks earlier. He notes pain and deformity about the ulnar aspect of his hand. The best
treatment option is
Figure 1 Figure 2 Figure 3

A. closed reduction and cast immobilization.


B. open reduction and internal fixation (ORIF).
C. arthrodesis.
D. resection arthroplasty.

Correct answer: B

 Discussion
Figure 4 Figure 5

The initial radiographs reveal a fourth and fifth carpometacarpal (CMC) joint fracture dislocation. The
injury is associated with a shear fracture of the dorsal rim of the hamate. Further assessment with CT
might be helpful in fully evaluating the extent of injury. Extensor carpi ulnaris is a deforming force at
the base of the fifth metacarpal. This unstable fracture dislocation could be treated with closed reduction
and pinning if the patient presented within a few days of injury. However, because he presented in a
delayed fashion (3 weeks after injury), open reduction with internal fixation was required (Figure 4 and
5). In the series by Zhang and associates, patients with fourth and fifth CMC fracture dislocations
presenting in a delayed fashion and treated nonsurgically had suboptimal results. Therefore, closed
reduction and casting are not appropriate. An arthrodesis and resection arthroplasty are salvage
procedures considered for a painful arthritic joint and would less likely should not be considered for this
acute injury.

 Recommended Readings
Zhang C, Wang H, Liang C, Yu W, Li Y, Shang R, Huang C, Huang C. The Effect of Timing on the
Treatment and Outcome of Combined Fourth and Fifth Carpometacarpal Fracture Dislocations. J Hand
Surg Am. 2015 Nov;40(11):2169-2175.e1. doi: 10.1016/j.jhsa.2015.07.017. Epub 2015 Sep 9. PubMed

Day CS, Stern PJ. Fractures of the metacarpals and phalanges. In: Wolfe SW, Hotchkiss RN, Pederson
WC, Kozin SH. Green's Operative Hand Surgery. Vol 1. 6th ed. Philadelphia, PA: Elsevier; 2011:239-
290

Question 30 of 100

Figures 1 through 3 show the clinical photographs obtained from a 45-year-old woman who is right-hand
dominant. She has pain in the left ring proximal interphalangeal (PIP) joint that gets worse during lifting or
gripping activities. On examination, she has PIP range of motion of 15° to 50° with laxity of the radial collateral
ligament and tenderness around the joint. The flexor and extensor tendons are intact. She has rotational
malalignment when making a composite fist. Radiographs reveal end-stage arthritis at the PIP joint. She elects to
move forward with surgery and undergoes arthroplasty. What component of the examination is essential to
determine which implant arthroplasty—silicone or surface replacement—is best?

Figure 1 Figure 2 Figure 3

A. Preoperative range of motion


B. Flexor tendon integrity
C. Rotational malalignment
D. Collateral ligament stability

Correct answer: D

 Discussion

This patient has end-stage arthritis in conjunction with ligament insufficiency. The treatment for arthritis
is arthroplasty or fusion. Given that her ring finger is affected, arthroplasty is recommended to preserve
motion and grip. Two types of arthroplasties are available: silicone and surface replacement. The
prerequisites are the same for both and include good bone stock, good sensibility of the joint, adequate
soft-tissue coverage, and normally functioning tendons. Adequate collateral ligaments are required for
surface replacement arthroplasty. This patient has a deficiency of the radial collateral ligament,
evidenced by her clinical examination. Thus, silicone arthroplasty is the recommended option for joint
replacement in this patient.

 Recommended Readings

Yamamoto M, Chung KC. Implant Arthroplasty: Selection of Exposure and Implant. Hand Clin. 2018
May;34(2):195-205. doi: 10.1016/j.hcl.2017.12.009. PubMed

Criner KT, Ilyas AM. Silicone arthroplasty for chronic proximal interphalangeal joint dislocations. Tech
Hand Up Extrem Surg. 2011 Dec;15(4):209-14. doi: 10.1097/BTH.0b013e31820f8b53. PubMed

Bales JG, Wall LB, Stern PJ. Long-term results of Swanson silicone arthroplasty for proximal
interphalangeal joint osteoarthritis. J Hand Surg Am. 2014 Mar;39(3):455-61. doi:
10.1016/j.jhsa.2013.11.008. PubMed

Srnec JJ, Wagner ER, Rizzo M. Implant Arthroplasty for Proximal Interphalangeal,
Metacarpophalangeal, and Trapeziometacarpal Joint Degeneration. J Hand Surg Am. 2017
Oct;42(10):817-825. doi: 10.1016/j.jhsa.2017.07.030. Epub 2017 Aug 30. PubMed

Question 31 of 100
Figures 1 through 4 are the wrist MR images of a 43-year-old right-hand-dominant bricklayer who reports
gradually progressive left hand weakness for 4 months. He describes difficulty gripping objects, tying his shoes,
and holding utensils. He denies any numbness, paresthesias, or a previous injury. An examination reveals intact
sensation in a median, radial, and ulnar nerve distribution. He has atrophy of hand interossei and a positive
Froment sign finding. He has no Tinel sign finding at the wrist or elbow and no exacerbation of symptoms with
elbow hyperflexion. Electromyography shows signs of denervation in an ulnar nerve distribution distal to the
wrist. What is the best next step?

Figure 1 Figure 2

Figure 3 Figure 4

A. Cubital tunnel release


B. Guyon's canal release
C. Hook-of-hamate excision
D. Excision of the ganglion cyst

Correct answer: D
 Discussion

The MR images show a lesion consistent with a ganglion cyst located near the hook of the hamate. The
ulnar nerve divides into motor and sensory branches just proximal to this lesion. In this case, the
ganglion cyst compresses the ulnar nerve motor branch but not the sensory branch, resulting in motor
dysfunction but no sensory disturbance. Excision of the ganglion cyst should alleviate his symptoms.

Compression of the ulnar nerve proximal to the motor branch take-off (in either the cubital tunnel or
proximal Guyon’s canal) would cause both sensory and motor dysfunction. Although chronic nonunion
of the hook of the hamate can cause ulnar nerve symptoms, the hook of the hamate appears intact on the
MR image. The MR image shows a lesion that is well circumscribed with high intensity on T1 and T2
images, consistent with a benign ganglion cyst, and ganglion cysts are relatively common lesions in this
area.

 Recommended Readings

Maroukis BL, Ogawa T, Rehim SA, Chung KC. Guyon canal: the evolution of clinical anatomy. J Hand
Surg Am. 2015 Mar;40(3):560-5. doi: 10.1016/j.jhsa.2014.09.026. Epub 2014 Oct 29. PubMed

Wang B, Zhao Y, Lu A, Chen C. Ulnar nerve deep branch compression by a ganglion: a review of nine
cases. Injury. 2014 Jul;45(7):1126-30. doi: 10.1016/j.injury.2014.03.017. Epub 2014 Apr 3. PubMed

Question 32 of 100

Figures 1 and 2 are the radiographs of a 55-year-old woman homemaker with a 1-year history of insidious onset
left wrist pain. She has failed conservative treatment and desires surgery. Her medical history is complicated by
a smoking history of 1.5 packs of cigarettes per day. At the time of surgery her capitate articular surface is
normal in appearance. The best procedure for her would be
Figure 1 Figure 2

A. radial shortening osteotomy.


B. capitate shortening osteotomy.
C. scaphoid excision and four-corner fusion.
D. proximal row carpectomy.

Correct answer: D

 Discussion
This patient has Lichtman stage 3B Kienbock disease. She is 55 years old and is a "low-demand"
patient; however, she is a heavy smoker. Based on her condition and her current smoking status, salvage
treatment that does not require bone healing such as a proximal row carpectomy is likely the best
treatment option. A radial shortening osteotomy and a capitate shortening osteotomy may be helpful in
offloading the lunate, but both procedures require bone healing and are better options in earlier stages of
Kienbock disease. A scaphoid excision and four-corner fusion is typically performed for scapholunate
advanced collapse or scaphoid nonunion advanced collapse wrist arthritis and would not be
recommended in this scenario, as the lunate is avascular.

 Recommended Readings

Diao E, Andrews A, Beall M. Proximal row carpectomy. Hand Clin. 2005 Nov;21(4):553-9. PubMed

Lichtman DM, Lesley NE, Simmons SP. The classification and treatment of Kienbock's disease: the
state of the art and a look at the future. J Hand Surg Eur Vol. 2010 Sep;35(7):549-54. doi:
10.1177/1753193410374690. Epub 2010 Jul 9. PubMed

Question 33 of 100

Figure 1 shows the clinical photograph obtained from a child with a congenital difference of the hand. What
clinical feature(s) is/are characteristic of this condition?
Figure 1

A. Cardiac anomalies
B. Radial deviation of the thumb
C. Acrosyndactyly with proximal sinus tracts
D. Absence of the ulna

Correct answer: C

 Discussion

The clinical photograph reveals a child with amniotic band syndrome or constriction band syndrome. If
a band causes an autofusion of the digits without amputation, acrosyndactyly can occur, as demonstrated
in the clinical photograph. Typically, a proximal sinus tract with a distal syndactyly is present.

Radial deviation of the thumb can be seen most frequently in Apert syndrome. Cardiac anomalies are
associated with many congenital upper extremity differences but are not characteristic of amniotic band
syndrome. Ulnar longitudinal deficiency is characterized by hypoplasia or complete absence of the ulna.

 Recommended Readings

Kawamura K, Chung KC. Constriction band syndrome. Hand Clin. 2009 May;25(2):257-64. doi:
10.1016/j.hcl.2008.10.007. PubMed

Koskimies E, Syvänen J, Nietosvaara Y, Mäkitie O, Pakkasjärvi N. Congenital constriction band


syndrome with limb defects. J Pediatr Orthop. 2015 Jan;35(1):100-3. doi:
10.1097/BPO.0000000000000206. PubMed

Waters PM, Bae DS. Aphalangia and amniotic band syndrome. In:Waters PM, Bae DS, eds. Pediatric
Hand and Upper Limb Surgery: A Practical Guide. 1st ed. Philadelphia, PA: Lippincott, Williams &
Wilkins; 2012:81-91.

Moran SL, Jensen M, Bravo C. Amniotic band syndrome of the upper extremity: diagnosis and
management. J Am Acad Orthop Surg. 2007 Jul;15(7):397-407. Full text
Question 34 of 100

Botulinum toxin is used to treat vasospastic disorders of the hand such as the Raynaud phenomenon to improve
digital perfusion and reduce pain. Botulinum toxin enables which transmitter to be unopposed, resulting in
vasodilation?

A. Substance P
B. Glutamate
C. Rho kinase
D. Nitric oxide

Correct answer: D

 Discussion

Nitric oxide is the only transmitter listed that is not inhibited by botulinum toxin. Substance P and
glutamate are inhibited by botulinum toxin from release by pain nociceptors, thus reducing pain.
Fonseca and associates have postulated that botulinum toxin inhibits the RhoA kinase pathway by
blocking reactive oxygen species, which in turn does not allow actin/myosin to activate, thus preventing
vasoconstriction of smooth muscle. Blocking the RhoA kinase pathway allows the action of nitric oxide
to be unopposed, causing vasodilation. Nitric oxide is a potent vasodilator. Thus, botulinum toxin
promotes nitric oxide activity to increase vasodilation.

 Recommended Readings

Neumeister MW. The role of botulinum toxin in vasospastic disorders of the hand. Hand Clin. 2015
Feb;31(1):23-37. doi: 10.1016/j.hcl.2014.09.003. Epub 2014 Nov 25. PubMed

Fonseca C, Abraham D, Ponticos M. Neuronal regulators and vascular dysfunction in Raynaud's


phenomenon and systemic sclerosis. Curr Vasc Pharmacol. 2009 Jan;7(1):34-9. PubMed
Question 35 of 100

Figures 1 and 2 depict the postoperative radiographs obtained from a 22-year-old man who was involved in a
motor vehicle accident. The most likely limitation in motion arising from this treatment is
Figure 1 Figure 2

A. loss of wrist flexion.


B. loss of wrist extension.
C. loss of elbow extension.
D. loss of pronation.

Correct answer: D

 Discussion

This patient sustained fractures of his radius and ulna; both were treated with plate and screw fixation.
The plate used on the radius was straight, resulting in loss of the radial bow, which is critical for
enabling the radius to curve around the ulna during pronation. This patient is unable to pronate beyond
20°.

Schemitsch and Richards correlated a good functional outcome, defined as more than 80% of normal
rotation of the forearm, with restoration of the normal amount and location of the radial bow.
Additionally, they related the restoration of grip strength with appropriate restoration of the radial bow.
Matthews and associates reported little significant loss of rotation with 10° of angulation; however, 20°
of angulation resulted in a statistically and clinically significant loss of forearm rotation.

 Recommended Readings

Matthews LS, Kaufer H, Garver DF, Sonstegard DA. The effect on supination-pronation of angular
malalignment of fractures of both bones of the forearm. J Bone Joint Surg Am. 1982 Jan;64(1):14-7.
PubMed

Schemitsch EH, Richards RR. The effect of malunion on functional outcome after plate fixation of
fractures of both bones of the forearm in adults. J Bone Joint Surg Am. 1992 Aug;74(7):1068-78.
PubMed
Question 36 of 100

Figure 1 is the ultrasound of a 23-year-old patient who has had a volar radial 1.5-cm tender and painful wrist
mass for 6 months. The additional workup prior to surgery should consist of

Figure 1

A. serum and urine protein electrophoresis.


B. a chest CT scan.
C. MRI with intravenous contrast.
D. age-appropriate presurgical laboratory studies.

Correct answer: D

 Discussion

The ultrasound shows a homogeneous anechoic mass consistent with a ganglion cyst. As a benign
lesion, no further workup or biopsy is required prior to a marginal surgical excision other than age-
appropriate laboratory studies. An MRI study with contrast would provide no diagnostic benefit.

 Recommended Readings
Mayerson JL, Scharschmidt TJ, Lewis VO, Morris CD. Diagnosis and Management of Soft-tissue
Masses. J Am Acad Orthop Surg. 2014 Nov;22(11):742-50. doi: 10.5435/JAAOS-22-11-742. Full text

Amrami KK, Bishop AT, Berger RA: Radiology Corner: Imaging Soft-Tissue Tumors of the Hand and
Wrist: Case Presentation and Discussion, Journal of the American Society for Surgery of the Hand,
2005;Volume 5(Issue 4):186-192. http://www.jhandsurg.org/article/S1531-0914(05)00149-X/abstract.
Last accessed 12-14-2018

Question 37 of 100

A 65-year-old woman has severe pain and numbness in her hand. She notes frequent awakenings at nighttime
and difficulty with fine tasks. She also has a history of cervical radiculopathy and notes intermittent pain in her
upper arm and periscapular region. An examination reveals a positive Tinel sign over the midforearm and carpal
tunnel. Electrodiagnostic testing shows a median nerve sensory distal latency of 3.8 ms (normal latency is 3.5
ms). Which intervention or test would best predict if carpal tunnel release would be successful in relieving this
patient's symptoms?

A. Trigger point injections with lidocaine


B. Carpal tunnel corticosteroid injection
C. Ultrasound of the wrist
D. Carpal tunnel view radiograph

Correct answer: B

 Discussion

This patient demonstrates several upper extremity issues including possible carpal tunnel syndrome,
cervical radiculopathy, and pronator syndrome. The electrodiagnostic testing is equivocal, and a
corticosteroid carpal tunnel injection should be performed prior to surgical intervention to assess its
effectiveness in eliminating the patient's symptoms. Positive response (meaning improvement in
symptoms), after corticosteroid injection at the carpal tunnel correlates well with symptom relief
following surgery. Trigger-point injections are not indicated for carpal tunnel syndrome. Ultrasound and
carpal tunnel view radiograph can provide diagnostic information but would not be helpful in
determining treatment in this specific case.
 Recommended Readings

Kane PM, Daniels AH, Akelman E. Double Crush Syndrome. J Am Acad Orthop Surg. 2015
Sep;23(9):558-62. Full text

Ponnappan RK, Khan M, Matzon JL, Sheikh ES, Tucker BS, Pepe MD, Tjoumakaris FP, Nassr AN.
Clinical Differentiation of Upper Extremity Pain Etiologies. J Am Acad Orthop Surg. 2015
Aug;23(8):492-500. Full text

Question 39 of 100

Figures 1 through 5 show the radiographs obtained from a 37-year-old man who has a 10-year history of right,
ulnar-sided wrist pain and a volar ulnar prominence with wrist supination. Approximately 20 years ago, he had a
forearm injury that was definitively treated in a long arm cast. What surgical treatment option is most likely to
improve his symptoms and maintain pronosupination?

Figure 1 Figure 2 Figure 3


Figure 4 Figure 5

A. Distal radial ulnar joint (DRUJ) ligament reconstruction


B. Ulnar head implant arthroplasty
C. Radial shaft osteotomy
D. One-bone forearm procedure
Correct answer: C

 Discussion

The patient sustained a radial shaft fracture with subsequent apex volar malunion. As a result, his distal
ulna subluxates volarly with wrist supination. Radiographs of the wrist reveal minimal arthritic changes.
The most appropriate treatment option is to surgically correct his radial shaft malunion, which would
indirectly address his DRUJ instability. A DRUJ ligament reconstruction or triangular fibrocartilage
complex repair could be used to augment DRUJ stability; however, they might be unnecessary after
correction of the radial shaft malunion. A DRUJ ligament reconstruction alone would not achieve
stability of the DRUJ joint and maintain full wrist pronosupination. An ulnar head implant arthroplasty
would not be reliable in eliminating the instability or the pain. Similarly, a one-bone forearm procedure
might improve the patient's pain and instability but at the cost of abnormal wrist and forearm mechanics
and kinematics.

 Recommended Readings

Miller A, Lightdale-Miric N, Eismann E, Carr P, Little KJ. Outcomes of Isolated Radial Osteotomy for
Volar Distal Radioulnar Joint Instability Following Radial Malunion in Children. J Hand Surg Am. 2018
Jan;43(1):81.e1-81.e8. doi: 10.1016/j.jhsa.2017.07.012. Epub 2017 Aug 23. PubMed

Nishiwaki M, Welsh MF, Gammon B, Ferreira LM, Johnson JA, King GJ. Effect of Volarly Angulated
Distal Radius Fractures on Forearm Rotation and Distal Radioulnar Joint Kinematics. J Hand Surg Am.
2015 Nov;40(11):2236-42. doi: 10.1016/j.jhsa.2015.07.034. Epub 2015 Sep 26. PubMed

Question 40 of 100

Figures 1 and 2 are the radiographs of an 18-year-old man who had surgery 6 months ago at an outside
institution. He is being referred now because he has persistent pain. He is tender over the scaphoid at the
snuffbox. What is the most appropriate next imaging step in his pain workup?
Figure 1 Figure 2
A. MR imaging with contrast
B. MR imaging without contrast
C. CT scan along the scaphoid axis
D. Axial-cut CT scans with reformats

Correct answer: C

 Discussion

Scaphoid nonunions are difficult to diagnose on plain radiographs, which offer poor reliability when
attempting to determine if there is bridging trabeculae crossing the fractures site. CT scans are more
useful for diagnosing scaphoid nonunion. When scanned using conventional axial cuts, the slices
mayskip through the fracture nonunion site, thereby missing the defect, even with reformats. MR
imaging is useful in diagnosing acute scaphoid fractures and has a high sensitivity and diagnostic value
for excluding scaphoid fractures as well. Contrast does not enhance the utility of MR imaging in fracture
diagnosis.

 Recommended Readings

Yin ZG, Zhang JB, Kan SL, Wang XG. Diagnosing suspected scaphoid fractures: a systematic review
and meta-analysis. Clin Orthop Relat Res. 2010 Mar;468(3):723-34. doi: 10.1007/s11999-009-1081-6.
Epub 2009 Sep 15. PubMed

Yin ZG, Zhang JB, Gong KT. Cost-Effectiveness of Diagnostic Strategies for Suspected Scaphoid
Fractures. J Orthop Trauma. 2015 Aug;29(8):e245-52. doi: 10.1097/BOT.0000000000000316. PubMed

Lutsky K, Matzon JL. Persistent fracture line after scaphoid fracture fixation. J Hand Surg Am.
2014 Nov;39(11):2294-6. doi: 10.1016/j.jhsa.2014.08.030. Epub 2014 Oct 3. PubMed

Ring D, Jupiter JB, Herndon JH. Acute fractures of the scaphoid. J Am Acad Orthop Surg. 2000 Jul-
Aug;8(4):225-31. Full text
Question 41 of 100

A 25-year-old man sustains a left brachial plexus injury from a fall while rock climbing. Examination reveals poor
intrinsic function of the hand, ptosis, and miosis. He is able to abduct and forward flex his shoulder with full
strength. This combination of physical findings is most suggestive of what pattern of nerve injury?

A. C5-C6 postganglionic injury


B. C8-T1 preganglionic injury
C. C5 through C7 preganglionic injury
D. C8-T1 postganglionic injury

Correct answer: B

 Discussion

A preganglionic lesion occurs proximal to the spinal foramen, whereas a postganglionic lesion occurs
distal to the spinal foramen in the root, trunk, division, cord, or branches of the brachial plexus. The
Horner sign, which is characterized by miosis, ptosis, anhydrosis, and enophthalmos, results from an
injury to the sympathetic ganglion, which lies in close proximity to the T1 root level. The presence of a
Horner sign is highly suggestive of a T1 preganglionic injury. Other physical examination indicators of a
preganglionic injury include atrophy of the parascapular muscles (injury to the dorsal rami of the
cervical spinal nerve roots), winged scapula (injury to the long thoracic nerve) and hemidiaphragmatic
paralysis (phrenic nerve injury).

The lack of intrinsic hand function in this patient is also suggestive of an injury at the level of C8-T1.
Preservation of shoulder abduction and forward flexion would not typically be seen with an injury to the
C5-C6 roots or the upper trunk.

 Recommended Readings

Giuffre JL, Kakar S, Bishop AT, Spinner RJ, Shin AY. Current concepts of the treatment of adult
brachial plexus injuries. J Hand Surg Am. 2010 Apr;35(4):678-88; quiz 688. doi:
10.1016/j.jhsa.2010.01.021. Review. Erratum in: J Hand Surg Am. 2010 Jul;35(7):1226. Kakar, Sanjiv
[corrected to Kakar, Sanjeev]. PubMed
O'Shea K, Feinberg JH, Wolfe SW. Imaging and electrodiagnostic work-up of acute adult brachial
plexus injuries. J Hand Surg Eur Vol. 2011 Nov;36(9):747-59. doi: 10.1177/1753193411422313. Epub
2011 Sep 15. PubMed

Limthongthang R, Bachoura A, Songcharoen P, Osterman AL. Adult brachial plexus injury: evaluation
and management. Orthop Clin North Am. 2013 Oct;44(4):591-603. doi: 10.1016/j.ocl.2013.06.011.
Epub 2013 Sep 6. PubMed

Question 42 of 100

Based on the best available evidence, what is the maximum number of days at which a successful manipulation
can be performed following collagenase injection?

A. 7
B. 5
C. 3
D. 1

Correct answer: A

 Discussion

In the original clinical trial, the authors performed manipulation at 24 to 72 hours following the
collagenase injection for Dupuytren disease. Evidence now suggests that up to 7 days is a safe and well-
tolerated interval for manipulation following collagenase injection.

 Recommended Readings

Badalamente MA, Hurst LC, Benhaim P, Cohen BM. Efficacy and safety of collagenase clostridium
histolyticum in the treatment of proximal interphalangeal joints in dupuytren contracture: combined
analysis of 4 phase 3 clinical trials. J Hand Surg Am. 2015 May;40(5):975-83. doi:
10.1016/j.jhsa.2015.02.018. Epub 2015 Apr 2. PubMed
Mickelson DT, Noland SS, Watt AJ, Kollitz KM, Vedder NB, Huang JI. Prospective randomized
controlled trial comparing 1- versus 7-day manipulation following collagenase injection for dupuytren
contracture. J Hand Surg Am. 2014 Oct;39(10):1933-1941.e1. doi: 10.1016/j.jhsa.2014.07.010. Epub
2014 Sep 4. PubMed

Question 43 of 100

Figure 1 is the radiograph of a 22-year-old man who underwent an open reduction and pinning of a perilunate
dislocation 10 weeks ago. The hardware has been removed. What is the best next step?

Figure 1

A. Observation
B. Vascularized bone grafting to the lunate
C. Core decompression of the radius and ulna
D. Immobilization

Correct answer: A

 Discussion
Lunate or perilunate dislocations are usually treated with open reduction and internal fixation through a
dorsal or combined dorsal and volar approach. A high index of suspicion is necessary when treating
patients who sustain multiple trauma because as many as 25% of lunate or perilunate dislocations are
missed initially.

The radio dense appearance of the lunate seen in Figure 1 is an example of transient ischemia of the
lunate that can occur following treatment of lunate and perilunate dislocations. It has been reported in up
to 12.5% of cases. This usually is seen between 1 and 4 months post injury with a relative radio density
of the lunate. This appearance of the lunate should not be over treated and usually is a benign self-
limiting event. Surgery is not indicated at this time; the incorrect responses are treatment options for
Kienbock disease.

Treatment of the lunate or perilunate dislocation involves initial gentle closed reduction followed by
open reduction, ligamentous and bone repair, and internal fixation. Median nerve dysfunction is
common, and a simultaneous carpal tunnel release is often performed. Early treatment seems to produce
better results, but good results have been reported when treatment is delayed for up to 6 months. The
many questions regarding treatment of this problem involve the use of capsulodesis to supplement
intercarpal ligament repair, repair/stabilization of the lunotriquetral interval vs no treatment of that
articulation, and intercarpal fixation techniques. In delayed cases, proximal row carpectomy when the
head of the capitate is intact and total wrist fusion if there are degenerative changes have been used.
Chondral injuries are common, may not be recognized on radiographs, and may negatively affect long-
term outcomes. Even when treatment is optimal, this injury is associated with a guarded prognosis and
possible permanent partial loss of wrist motion and grip strength. At 10-year follow-up, radiographs will
often demonstrate degenerative changes, but these changes do not always substantially negatively affect
hand function.

 Recommended Readings

Forli A, Courvoisier A, Wimsey S, Corcella D, Moutet F. Perilunate dislocations and transscaphoid


perilunate fracture-dislocations: a retrospective study with minimum ten-year follow-up. J Hand Surg
Am. 2010 Jan;35(1):62-8. doi: 10.1016/j.jhsa.2009.09.003. Epub 2009 Nov 22. PubMed

Vitale MA, Seetharaman M, Ruchelsman DE. Perilunate dislocations. J Hand Surg Am. 2015
Feb;40(2):358-62; quiz 362. doi: 10.1016/j.jhsa.2014.10.006. Epub 2014 Nov 15. PubMed

White RE Jr, Omer GE Jr. Transient vascular compromise of the lunate after fracture-dislocation or
dislocation of the carpus. J Hand Surg Am. 1984 Mar;9(2):181-4. PubMed

Stanbury SJ, Elfar JC. Perilunate dislocation and perilunate fracture-dislocation. J Am Acad Orthop
Surg. 2011 Sep;19(9):554-62. Full text
Question 44 of 100

A 64-year-old woman with rheumatoid arthritis cannot fully extend her fingers actively at the
metacarpophalangeal (MCP) level. Full passive extension is possible, but she cannot actively maintain that
extension when her fingers are released. The MCP joints do extend when her wrist is passively flexed. What is
the most likely cause of this problem?

A. Extensor tendon ruptures at the wrist


B. Subluxation of the extensor mechanisms at the MCP joint
C. Caput ulnae syndrome
D. Posterior interosseous nerve palsy

Correct answer: D

 Discussion

Loss of MCP extension is common in the setting of rheumatoid arthritis, but potential causes are varied.
An understanding of various etiologies is needed to address the multiple facets of inflammatory
arthropathy, for which causes include extensor tendon rupture, MCP joint flexion
contracture/dislocation, subluxation of the extensor tendons at the MCP joint (sagittal band rupture), and
posterior interosseous nerve (PIN) palsy. These causes are differentiated during an examination.

Extensor tendon ruptures, as seen with caput ulnae syndrome (Vaughan-Jackson syndrome) allow for
passive MCP extension, not active extension or the ability to maintain extension actively, and would not
involve finger extension with passive flexion tenodesis of the wrist. Flexion contractures of the MCP
joints, as may be seen in dislocations, would not allow active or passive motion.

Subluxation of the extensor tendons at the MCP joints, as seen in attenuation of the sagittal bands, will
usually preclude the initiation of active extension, but patients will be able to maintain extension of the
MCP joints once they are placed in extension by the examiner. With a PIN palsy, no active extension or
active hold is possible, but passive extension and tenodesis with wrist flexion are maintained. Synovitis
of the elbow is the most likely cause of the PIN palsy. An extensor tenosynovitis or caput ulnae
syndrome would not prohibit MCP extension unless the condition progressed to the point of causing
other pathology.
 Recommended Readings

Steichen JB, Christensen AW. Posterior interosseous nerve compression syndrome. In: Gelberman RH
ed. Operative Nerve Repair and Reconstruction. Philadelphia, PA: JB Lippincott; 1991:1151-1157

Millender LH, Nalbuff EA, Holdsworth DE. Posterior interosseous nerve syndrome secondary to
rheumatoid synovitis. J Bone Joint Surg Am. 1973 Mar; 55(2):375-7. PubMed

Question 45 of 100

The development of complex regional pain syndrome (CRPS) following distal radius fracture is associated with
what factor?

A. Diabetes
B. Fibromyalgia
C. Nonsurgical fracture management
D. Male gender

Correct answer: B

 Discussion

CRPS is an uncommon complication following distal radius fractures; its incidence is reported to range
between 1% and 37%. Two recent studies have evaluated for risk factors in the development of CRPS
following distal radius fractures. Female gender, concomitant fracture of the distal ulna, and surgical
treatment were all associated with an increased likelihood of CRPS, as was fibromyalgia. Older age was
identified as conferring both an increased and a decreased risk for CRPS in the two studies.

 Recommended Readings
Crijns TJ, van der Gronde BATD, Ring D, Leung N. Complex Regional Pain Syndrome After Distal
Radius Fracture Is Uncommon and Is Often Associated With Fibromyalgia. Clin Orthop Relat Res. 2018
Apr;476(4):744-750. doi: 10.1007/s11999.0000000000000070. PubMed

Lipman MD, Hess DE, Werner BC, Deal DN. Fibromyalgia as a Predictor of Complex Regional Pain
Syndrome After Distal Radius Fracture. Hand(N Y). 2017 Oct 1:1558944717735949. doi:
10.1177/1558944717735949. [Epub ahead of print] PubMed

Question 46 of 100

A 20-year-old woman with spastic hemiplegia is evaluated for function and hygiene issues with her right wrist.
Her wrist has a resting posture of 90° of flexion and can be passively extended to 65° of flexion. Her fingers are
flexed into her palm but can be passively extended with the wrist at 95°. What treatment is likely to provide the
most durable result for improved hygiene, function, and cosmesis?

A. Flexor carpi ulnaris to extensor carpi radialis brevis transfer


B. Fractional lengthening of the wrist and finger flexor tendons
C. Wrist arthrodesis with proximal row carpectomy
D. Botulinum toxin injection

Correct answer: C

 Discussion

The patient has a static deformity of the wrist with a fixed flexion deformity of more than 45°. Soft-
tissue procedures such as those referenced in options A and B would not be sufficient to address the
degree of contracture. Additionally, Botulinum toxin injections would not provide relief for capsular
contractures. Wrist arthrodesis combined with a proximal row carpectomy has been shown to provide a
functional and cosmetic alternative for patients with severe wrist flexion contractures.

 Recommended Readings
Hartigan BJ, Nagle DJ, Foley MJ. Wrist arthrodesis with excision of the proximal carpal bones using the
Ao/ASIF wrist fusion plate and local bone graft. J Hand Surg Br. 2001 Jun;26(3):247-51. PubMed

Carlson MG, Athwal GS, Bueno RA. Treatment of the wrist and hand in cerebral palsy. J Hand Surg
Am. 2006 Mar;31(3):483-90. PubMed

Van Heest AE, Strothman D. Wrist arthrodesis in cerebral palsy. J Hand Surg Am. 2009
Sep;34(7):1216-24. doi: 10.1016/j.jhsa.2009.03.006. Epub 2009 Jun 4. PubMed

Donadio J, Upex P, Bachy M, Fitoussi F. Wrist arthrodesis in adolescents with cerebral palsy. J Hand
Surg Eur Vol. 2016 Sep;41(7):758-62. doi: 10.1177/1753193415625611. Epub 2016 Jan 14. PubMed

Question 47 of 100

Figures 1 and 2 are the radiographs of a 36-year-old right-hand-dominant man who has had persistent wrist pain
for 6 months after a motor vehicle collision. The initial treatment was splint immobilization. What is the best
next step?
Figure 1 Figure 2

A. Therapy/rehabilitation
B. Open reduction and internal fixation (ORIF)
C. Proximal row carpectomy
D. Wrist arthrodesis

Correct answer: C

 Discussion

This patient has a chronic untreated volar lunate dislocation. Lunate dislocations are usually the result of
a high-energy injury. Recommended treatment for an acute lunate dislocation is ORIF with repair of
injured structures (ligament and bone). If the patient has paresthesias in a median nerve distribution,
carpal tunnel release is recommended in the same setting as ORIF. Six months after injury, the prognosis
for successful ORIF is poor and proximal row carpectomy is recommended. Among perilunate/lunate
dislocations, 25% are initially missed. If a patient arrives for treatment and there is evidence of
radiocarpal and midcarpal arthrosis, wrist arthrodesis is recommended.

 Recommended Readings

Budoff JE. Treatment of acute lunate and perilunate dislocations. J Hand Surg Am. 2008
Oct;33(8):1424-32. doi: 10.1016/j.jhsa.2008.07.016. Review. PubMed

Stanbury SJ, Elfar JC. Perilunate dislocation and perilunate fracture-dislocation. J Am Acad Orthop
Surg. 2011 Sep;19(9):554-62. Full text

Question 48 of 100

Figures 1 through 3 show the MRI images and a radiograph obtained from a 31-year-old woman who has a 1-
year history of diffuse right wrist pain that is gradually worsening. She denies fever or chills and also denies a
history of injury. Her examination reveals no swelling, no erythema, an 80 degree arc of active wrist flexion and
extension, and dorsal wrist tenderness. The most likely diagnosis is

Figure 1 Figure 2 Figure 3

A. scapholunate advanced collapse (SLAC) wrist with cystic capitate changes.


B. idiopathic avascular necrosis (AVN) of the capitate.
C. capitate osteomyelitis.
D. aneurysmal bone cyst in the capitate.

 Discussion

This patient's history, examination, and imaging suggest a rare condition known as capitate AVN. SLAC
arthritis would show osteoarthritic changes in the radioscaphoid joint. Capitate osteomyelitis would
likely have more diffuse edematous changes on MRI and less range of motion secondary to associated
septic arthritis. An aneurysmal bone cyst in the capitate would likely show fluid-fluid levels on the MRI.

The treatment of capitate AVN is usually surgical and varies on the extent of the area of necrosis, the
degree of fragmentation, the degree of collapse, and the absence or presence of arthritic changes.
Common treatment options include vascularized bone grafts, resection of the affected area with
interposition tendon graft, intercarpal fusion, and complete wrist fusion or replacement.

 Recommended Readings
Kazmers NH, Rozell JC, Rumball KM, Kozin SH, Zlotolow DA, Levin LS. Medial Femoral Condyle
Microvascular Bone Transfer as a Treatment for Capitate Avascular Necrosis: Surgical Technique and
Case Report. J Hand Surg Am. 2017 Oct;42(10):841.e1-841.e6. doi: 10.1016/j.jhsa.2017.04.006. Epub
2017 May 9. PubMed

Peters SJ, Degreef I, De Smet L. Avascular necrosis of the capitate: report of six cases and review of the
literature. J Hand Surg Eur Vol. 2015 Jun;40(5):520-5. doi: 10.1177/1753193414524876. Epub 2014
Feb 25. PubMed

Question 49 of 100

When treating a closed long finger central slip tendon rupture conservatively, what is the most appropriate plan
of care?

A. Splint the proximal interphalangeal (PIP) joint in flexion with early motion of the distal interphalangeal
(DIP) joint
B. Allow early motion of the PIP joint with DIP extension joint splinting
C. Splint both the PIP and DIP joints in full extension
D. Splint the PIP joint in extension with early motion of the DIP joint

Correct answer: D

 Discussion

Closed central slip injuries treated nonsurgically require extension splinting of the PIP joint. DIP joint
active range of motion is allowed during this time period. This allows the connections between the
lateral bands and the central slip to pull the central slip distally with DIP joint active motion, minimizing
the gap across the central tendon injury and keeping the DIP joint from getting stiff as well.

 Recommended Readings
Scott SC. Closed injuries to the extension mechanism of the digits. Hand Clin.2000 Aug;16(3):367-73,
viii. PubMed

Posner MA, Green SM. Diagnosis and treatment of finger deformities following injuries to the extensor
tendon mechanism. Hand Clin. 2013 May;29(2):269-81. doi: 10.1016/j.hcl.2013.03.003. PubMed

Question 50 of 100

Figure 1 is the clinical photograph of a 64-year-old man who crashed while riding his motorcycle. An
examination reveals his long-finger metacarpophalangeal (MP) joint is stuck in extension. He cannot passively or
actively flex at the MP joint. A hand radiograph is seen in Figure 2. Which interposed structure is preventing
reduction?

Figure 1 Figure 2

A. Flexor tendons
B. Lateral band
C. Lumbrical
D. Volar plate
Correct answer: D

 Discussion

This patient has a dorsally dislocated MP joint. In these cases, the volar plate can be displaced dorsal to
the metacarpal head, preventing reduction. Although early publications described a “noose effect” of the
lumbrical and flexor tendons, the primary block to reduction is the volar plate. Simple MP dislocations
can be reduced closed by flexing the wrist and then gently sliding the base of the proximal phalanx over
the end of the metacarpal. Longitudinal traction on the finger will only incarcerate the volar plate further
and should be avoided. Patients with complex dislocations that fail closed reduction require open
reduction.

 Recommended Readings

Bohart PG, Gelberman RH, Vandell RF, Salamon PB. Complex dislocations of the metacarpophalangeal
joint. Clin Orthop Relat Res. 1982 Apr;(164):208-10. PubMed

Afifi AM, Medoro A, Salas C, Taha MR, Cheema T. A cadaver model that investigates irreducible
metacarpophalangeal joint dislocation. J Hand Surg Am. 2009 Oct;34(8):1506-11. doi:
10.1016/j.jhsa.2009.06.001. Epub 2009 Aug 22. PubMed

Question 51 of 100

Which method of flexor tendon repair that necessitates excursion through the A2 pulley allows for the most
thorough assessment of tendon gliding?

A. 4-strand repair with 6-0 epitendinous suture with Bier block anesthesia
B. 4-strand repair with 6-0 epitendinous suture under local anesthesia only
C. 6-strand repair with regional anesthesia
D. Repair of the flexor tendon with incision of the remaining A2 pulley

Correct answer: B
 Discussion

Wide-awake repair under only local anesthesia, regardless of the technique, allows direct inspection of
the tendon repair and active excursion. Regional anesthesia and Bier block anesthesia do not
allow active motion (Bier block necessitates continued use of a tourniquet, which limits muscle
function). The A2 pulley should be preserved, especially the distal 50%, to maintain tendon function.
All of the listed techniques for suture repair are acceptable options.

 Recommended Readings

Lalonde DH, Martin AL.Wide-awake flexor tendon repair and early tendon mobilization in zones 1 and
2. Hand Clin. 2013 May;29(2):207-13. doi: 10.1016/j.hcl.2013.02.009. Epub 2013 Mar 15. PubMed

Lalonde D. Minimally invasive anesthesia in wide awake hand surgery. Hand Clin. 2014 Feb;30(1):1-6.
doi: 110.1016/j.hcl.2013.08.015. Epub 2013 Nov 9. PubMed

Question 52 of 100

Figure 1 is the clinical photograph of a 42-year-old woman who has a lesion that has failed prior silver nitrate
applications. She experiences frequent bleeding from this lesion. A tissue biopsy performed by a dermatologist
revealed capillary hypertrophy with lobular arrangement. Which treatment is most appropriate to minimize
recurrence?
Figure 1

A. Sclerotherapy
B. Shave excision with cautery
C. Cryotherapy
D. Wide surgical excision

Correct answer: D

 Discussion

Figure 2 Figure 3

This lesion is a pyogenic granuloma, which is a common benign vascular lesion that can occur on skin
or mucosa. The etiology is unclear, although this lesion tends to occur in areas of physical trauma. Initial
treatment with silver nitrate with an average of 1.6 applications has a success rate of 85%. This patient,
however, has failed silver nitrate applications. Wide surgical excision (Figure 2 and 3) is associated with
the lowest recurrence rate and offers the benefit of a single procedure. Other options often necessitate
repeated procedures to completely eradicate this lesion.

 Recommended Readings
Quitkin HM, Rosenwasser MP, Strauch RJ. The efficacy of silver nitrate cauterization for pyogenic
granuloma of the hand. J Hand Surg Am. 2003 May;28(3):435-8. PubMed

Giblin AV, Clover AJ, Athanassopoulos A, Budny PG. Pyogenic granuloma – the quest for optimum
treatment: audit of treatment of 408 cases. J Plast Reconstr Aesthet Surg. 2007;60(9):1030-5. Epub 2007
May 2. PubMed

Question 53 of 100

In the injury shown in Figure 1 and 2, what ligament remains intact?


Figure 1 Figure 2

A. Short radiolunate
B. Scapholunate
C. Radioscaphocapitate
D. Dorsal radiocarpal
Correct answer: A

 Discussion

Perilunate dislocations result from high-energy injuries to the extended wrist. The injury shown is a
lunate dislocation. Two classification systems have been described, the Mayfield system and the
Herzberg system. Mayfield described the four stages of progressive ligamentous instability following
injury. In stage I, the radioscaphocapitate and scapholunate ligaments fail. Stage II involves dislocation
of the lunocapitate joint, usually a dorsal dislocation of the capitate. In stage III, the lunotriquetral
ligament fails. In stage IV, the dorsal radiocarpal ligament is torn, and the lunate dislocates volarly. The
short radiolunate ligament is the only ligament that remains intact, resulting in rotation of the lunate
volarly. Herzberg and associates further classified perilunate dislocations as stage I injuries and lunate
dislocations as stage II injuries. Lunate dislocations were further classified into stage IIA, in which the
lunate exhibits rotation less than 90°, and stage IIB, in which the lunate exhibits rotation greater than
90°. The radiographs represent a Mayfield stage IV, Herzberg stage IIA injury.

 Recommended Readings

Garcia-Elias M. Treatment of scapho-lunate instability. Ortop Traumatol Rehabil. 2006 Apr


28;8(2):160-8. PubMed

Mayfield JK, Johnson RP, Kilcoyne RK. Carpal dislocations: pathomechanics and progressive perilunar
instability. J Hand Surg Am. 1980 May;5(3):226-41. PubMed

Herzberg G, Comtet JJ, Linscheid RL, Amadio PC, Cooney WP, Stalder J. Perilunate dislocations and
fracture-dislocations: a multicenter study. J Hand Surg Am. 1993 Sep;18(5):768-79. PubMed

Question 54 of 100

A 50-year-old patient underwent multiple débridements for an open radial shaft fracture with bone loss. The
bed currently shows no evidence of infection but has a 14-cm diaphyseal bone defect. The most appropriate
treatment includes open reduction and internal fixation along with
A. free vascularized fibula.
B. calcium sulfate pellets.
C. corticocancellous autograft.
D. demineralized bone matrix.

Correct answer: A

 Discussion

The patient developed a large bone defect after undergoing multiple débridements for an open fracture.
The most appropriate graft in this setting is a vascularized bone graft. Considering the length of the
defect, a free vascularized fibular graft would be a suitable graft. The indications for a vascularized bone
graft include infection, inadequate vascularity of the surrounding tissues, bone defects larger than 6
centimeters, and previous failed bone grafts. The osteocytes survive in the vascularized graft, allowing
primary bone healing and thereby limiting a loss of graft strength. In contrast, nonvascularized bone
graft heals by creeping substitution with a loss of its initial strength. Calcium sulfate pellets and calcium
phosphate cement are synthetic bone substitutes with osteoconductive properties. Their role in fracture
healing is limited. Demineralized bone matrix is the matrix remaining after allograft undergoes
decalcification processing. These products alone would not be optimal in the treatment of such a large
bone defect.

 Recommended Readings

Noaman HH. Management of upper limb bone defects using free vascularized osteoseptocutaneous
fibular bone graft. Ann Plast Surg. 2013 Nov;71(5):503-9. doi: 10.1097/SAP.0b013e3182a1aff0.
PubMed

Adani R, Delcroix L, Innocenti M, Marcoccio I, Tarallo L, Celli A, Ceruso M. Reconstruction of large


posttraumatic skeletal defects of the forearm by vascularized free fibular graft. Microsurgery.
2004;24(6):423-9. PubMed

Question 55 of 100
A 45-year-old man underwent a fingertip amputation through the distal phalanx after his ring finger was caught
in a garage door. He was treated in the emergency department with a revision amputation by advancement of
the flexor digitorum profundus (FDP) tendon to the extensor mechanism. Three months following the injury, he
is able to fully flex his injured ring finger to touch his palm, but he reports that it is difficult for him to make a
tight fist due to decreased flexion of his other fingers. What is this complication called?

A. Lumbrical plus deformity


B. Intrinsic tightness
C. Quadrigia effect
D. Proximal interphalangeal joint contracture

Correct answer: C

 Discussion

The quadrigia effect can occur due to over-advancement of the FDP tendon during repair (usually
greater than 1 cm), development of FDP tendon adhesions, and (as in this case) "over the top" repair of
the FDP tendon to the extensor tendon after amputation at the distal phalanx level. All of these
conditions result in a functionally shortened FDP tendon of the injured digit. Because the FDP tendons
of the long, ring, and small digits share a common muscle belly, excursion of the combined tendons is
equal to the shortest tendon. Therefore, the uninjured digits will not have full excursion of their
respective FDP tendons and will not be able to close into a full fisting position. Treatment of this
condition is most commonly release of the injured FDP tendon.

A lumbrical plus deformity can occur in amputations distal to the flexor digitorum superficialis insertion
through the middle phalanx. The FDP tendon retracts and increases tension on the lumbrical muscle,
which leads to paradoxical interphalangeal (IP) joint extension with attempted flexion. Intrinsic
tightness and interphalangeal joint contractures can be caused by hand trauma but would not lead to the
clinical condition this patient has.

 Recommended Readings

Woo SH, Kim YW, Cheon HJ, Nam HJ, Kang DH, Kim JM, Ahn HC. Management of complications
relating to finger amputation and replantation. Hand Clin. 2015 May;31(2):319-38. doi:
10.1016/j.hcl.2015.01.006. Epub 2015 Feb 28. PubMed
Panattoni JB, De Ona IR, Ahmed MM. Reconstruction of fingertip injuries: surgical tips and avoiding
complications. J Hand Surg Am. 2015 May;40(5):1016-24. doi: 10.1016/j.jhsa.2015.02.010. Epub 2015
Mar 29. PubMed

Lee DH, Mignemi ME, Crosby SN. Fingertip injuries: an update on management. J Am Acad Orthop
Surg. 2013 Dec;21(12):756-66. Full text

Question 56 of 100

When performing a Green transfer for cerebral palsy—flexor carpi ulnaris (FCU) to extensor carpi radialis brevis
(ECRB)—in addition to improving wrist extension, what other motion may be improved if the FCU is routed
around the ulna instead of through the interosseous membrane?

A. Thumb extension
B. Forearm supination
C. Finger extension
D. Forearm pronation

Correct answer: B

 Discussion

The typical upper extremity deformity in spastic hemiplegic cerebral palsy consists of shoulder internal
rotation, elbow flexion, forearm pronation and wrist flexion, and ulnar deviation. The pronation position
of the forearm can make bimanual activities more challenging for the child. The wrist flexion and ulnar
deviation deformity interferes with finger function and therefore with grasp and release patterns. By
transferring the FCU tendon to the ECRB, the deforming force is released, and central wrist extension is
augmented. This transfer can lead to a supination moment when it is routed around the ulna to the ECRB
insertion on the dorsum of the wrist.

Thumb and finger extension are not affected by an FCU-to-ECRB tendon transfer. Forearm supination,
not pronation, is potentially improved with this tendon transfer.
 Recommended Readings

de Roode CP, James MA, Van Heest AE. Tendon transfers and releases for the forearm, wrist, and hand
in spastic hemiplegic cerebral palsy. Tech Hand Up Extrem Surg. 2010 Jun;14(2):129-34. doi:
10.1097/BTH.0b013e3181e3d785. PubMed

Patterson JM, Wang AA, Hutchinson DT. Late deformities following the transfer of the flexor carpi
ulnaris to the extensor carpi radialis brevis in children with cerebral palsy. J Hand Surg Am. 2010
Nov;35(11):1774-8. doi: 10.1016/j.jhsa.2010.07.014. PubMed

Cheema TA, Firoozbakhsh K, De Carvalho AF, Mercer D. Biomechanic comparison of 3 tendon


transfers for supination of the forearm. J Hand Surg Am. 2006 Dec;31(10):1640-4. PubMed

Question 57 of 100

Figure 1 shows a radiograph obtained from an active 30-year-old man who sustained an injury to his ring finger 1
week earlier. The most appropriate treatment is
Figure 1

A. open reduction and internal fixation (ORIF).


B. a mallet splint.
C. repair of the terminal tendon.
D. arthrodesis.

Correct answer: A

 Discussion
Figure 2

Figure 1 reveals evidence of an intra-articular distal phalanx fracture with a distal interphalangeal (DIP)
joint dorsal subluxation. This injury is unstable and requires surgical management for an active
individual. Volar distal phalanx fractures are often associated with flexor digitorum profundus avulsion
injuries, which are addressed concomitantly. This injury was treated with ORIF of the intra-articular
fracture, pinning of the DIP joint, and repair of an avulsed flexor digitorum profundus tendon with a
button on the dorsal nail plate, as shown in Figure 2.
Splint immobilization would not maintain a reduction of this unstable injury. The terminal tendon is not
injured in this patient but is often injured in a dorsal distal phalanx fracture with a volar dislocation.
Arthrodesis of the DIP is a salvage procedure and would not be considered acutely.

 Recommended Readings

Freilich AM. Evaluation and treatment of jersey finger and pulley injuries in athletes. Clin Sports Med.
2015 Jan;34(1):151-66. doi: 10.1016/j.csm.2014.09.001. Epub 2014 Nov 25. PubMed

Shah CM, Sommerkamp TG. Fracture dislocation of the finger joints. J Hand Surg Am. 2014
Apr;39(4):792-802. doi: 10.1016/j.jhsa.2013.10.001. PubMed

Question 58 of 100

A 55-year-old man was injured when a large piece of sheet metal lacerated his medial elbow while working at a
factory. He underwent primary repair of the lacerated structures shown in Figures 1 and 2 on the day of injury.
In addition to this surgical treatment, what nerve transfer procedure should be considered during this primary
operative intervention to improve his functional recovery?

Figure 1 Figure 2
A. Flexor digitorum superficialis (FDS) branch transfer to the extensor carpi radialis brevis (ECRB) branch
B. Third web space median fascicle transfer to the ulnar sensory fascicle
C. Flexor carpi ulnaris fascicle (FCU) transfer to the biceps branch
D. Terminal anterior interosseous nerve (AIN) transfer to the deep ulnar motor fascicle

Correct answer: D

 Discussion

In adults, the repair of high ulnar nerve injuries typically yields incomplete motor recovery and
disappointing functional results despite early surgical intervention and careful surgical technique. Early
transfer of the terminal branch of the AIN to the deep ulnar motor fascicle can rapidly reinnervate distal
targets and potentially preserve motor end plate function in the intrinsic musculature of the hand because
of the proximity of the nerve transfer to the target muscle.

Sensory deficits due to an ulnar nerve injury can be restored through a transfer of median sensory
fascicles to the distal ulna sensory fascicles. This procedure typically would not be considered at the
time of the original surgery, because sensory recovery is more likely than motor recovery in the setting
of a high ulnar nerve injury.

For radial nerve injuries, wrist extension can be restored through an FDS branch of the median nerve
transfer to the ECRB branch of the radial nerve. The FCU fascicle of the ulnar nerve can be transferred
to the biceps branch of the musculocutaneous nerve to restore elbow flexion and supination.

 Recommended Readings

Mackinnon SE. Donor Distal, Recipient Proximal and Other Personal Perspectives on Nerve Transfers.
Hand Clin. 2016 May;32(2):141-51. doi: 10.1016/j.hcl.2015.12.003. PubMed

Moore AM, Franco M, Tung TH. Motor and sensory nerve transfers in the forearm and hand. Plast
Reconstr Surg. 2014 Oct;134(4):721-30. doi: 10.1097/PRS.0000000000000509. PubMed

Woo A, Bakri K, Moran SL. Management of ulnar nerve injuries. J Hand Surg Am. 2015 Jan;40(1):173-
81. doi: 10.1016/j.jhsa.2014.04.038. Epub 2014 Nov 6. PubMed

Barbour J, Yee A, Kahn LC, Mackinnon SE. Supercharged end-to-side anterior interosseous to ulnar
motor nerve transfer for intrinsic musculature reinnervation. J Hand Surg Am. 2012 Oct;37(10):2150-9.
doi: 10.1016/j.jhsa.2012.07.022. PubMed
Question 59 of 100

What cardiac condition causes most upper extremity emboli?

A. Atrial fibrillation
B. Viral cardiomyopathy
C. Valvular disease
D. Atrial septal defect

Correct answer: A

 Discussion

Atrial fibrillation is responsible for approximately 80% of all upper extremity emboli. All other cardiac
conditions listed can cause upper extremity emboli; however, atrial fibrillation is the most common
cause.

Patients with an upper extremity embolic event should undergo prompt evaluation, with a careful history
and physical examination as well as focused laboratory tests for hypercoagulability. Arterial Doppler
studies or angiography is/are warranted. Electrocardiogram and echocardiogram are also used to
evaluate for potential cardiac abnormalities. Consultation with vascular, radiology, and cardiology
personnel is often necessary when patients present with upper extremity emboli. Treatment usually
involves anticoagulation, embolectomy if necessary, and treatment for any recognized cardiac
abnormality.

 Recommended Readings

Callum K, Bradbury A. ABC of arterial and venous disease: Acute limb ischaemia. BMJ. 2000 Mar
18;320(7237):764-7. Review. Erratum in: BMJ 2000 Apr 8;320(7240):984. PubMed

Zimmerman NB. Occlusive vascular disorders of the upper extremity. Hand Clin. 1993 Feb;9(1):139-50.
PubMed
Brinkley DM, Hepper CT. Heart in hand: structural cardiac abnormalities that manifest as acute
dysvascularity of the hand. J Hand Surg Am. 2010 Dec;35(12):2101-3. doi: 10.1016/j.jhsa.2010.09.005.
Epub 2010 Oct 23. PubMed

Question 60 of 100

Figures 1 through 4 are the radiographs, sagittal-cut CT scan, and coronal T1 MR image of a 16-year-old boy who
has wrist stiffness and pain after sustaining an injury 2 years ago. There is no bleeding from the proximal pole
during surgery. Which procedure will most likely result in restoration of alignment and healing?

Figure 1 Figure 2
Figure 3 Figure 4

A. 1,2 intercompartmental supraretinacular artery (ICSRA) graft


B. Free-vascularized medial femoral condyle graft
C. Iliac crest corticocancellous graft
D. 4+5 extensor compartmental artery (ECA) vascularized bone graft

Correct answer: B

 Discussion

The imaging studies show an established scaphoid waist nonunion with a humpback deformity
(significant flexion through the nonunion site) and carpal collapse. In addition, the proximal pole
appears sclerotic on the plain radiographs and appears poorly perfused on the MR image. Correction of
alignment of this scaphoid nonunion would require a volar approach with a structural bone graft.
Additionally, the graft would need to provide a vascular supply to the bone.

Both the 1,2 ICSRA (the Zaidenberg graft) and the 4+5 ECA grafts are vascularized grafts from the
dorsal distal radius. Neither of these grafts would correct the humpback deformity, and the 4+5 ECA
graft pedicle is not long enough to reach the scaphoid. An iliac crest bone graft could be used to correct
the deformity, but would not provide an adequate blood supply. A free-vascularized medial femoral
condyle graft provides both adequate bone graft to correct the deformity and revascularization of the
scaphoid.

 Recommended Readings

Jones DB Jr, Bürger H, Bishop AT, Shin AY. Treatment of scaphoid waist nonunions with an avascular
proximal pole and carpal collapse. A comparison of two vascularized bone grafts. J Bone Joint Surg Am.
2008 Dec;90(12):2616-25. doi: 10.2106/JBJS.G.01503. PubMed

Moon ES, Dy CJ, Derman P, Vance MC, Carlson MG. Management of nonunion following surgical
management of scaphoid fractures: current concepts. J Am Acad Orthop Surg. 2013 Sep;21(9):548-57.
doi: 10.5435/JAAOS-21-09-548. Full text

Question 61 of 100

Figure 1 is the clinical photograph of a 65-year-old right-hand dominant man who has finger contracture and
stiffness. He experiences minimal pain but has severe functional limitations and elects for treatment with
injectable collagenase Clostridium histolyticum. What types of collagen will be affected by this injection?
Figure 1

A. Types I and II
B. Types II and III
C. Types I and III
D. Types III and IV

Correct answer: C

 Discussion

Type II collagen is the predominant type found in articular cartilage. Type IV collagen is the
predominant type found in the basement membranes of neurovascular structures. Collagenase
Clostridium histolyticum injection targets type I and type III collagen.

 Recommended Readings

Chung L, Dinakarpandian D, Yoshida N, Lauer-Fields JL, Fields GB, Visse R, Nagase H. Collagenase
unwinds triple-helical collagen prior to peptide bond hydrolysis. EMBO J. 2004 Aug 4;23(15):3020-30.
Epub 2004 Jul 15. PubMed

Gaston RG, Larsen SE, Pess GM, Coleman S, Dean B, Cohen BM, Kaufman GJ, Tursi JP, Hurst LC.
The Efficacy and Safety of Concurrent Collagenase Clostridium Histolyticum Injections for 2
Dupuytren Contractures in the Same Hand: A Prospective, Multicenter Study. J Hand Surg Am. 2015
Oct;40(10):1963-71. doi: 10.1016/j.jhsa.2015.06.099. Epub 2015 Jul 26. PubMed

Desai SS, Hentz VR. The treatment of Dupuytren disease. J Hand Surg Am. 2011 May;36(5):936-42.
doi: 10.1016/j.jhsa.2011.03.002. PubMed

Question 62 of 100
After performing an uneventful partial palmar fasciectomy for Dupuytren contracture of the palm and ring
finger, a general postsurgical pain medication prescription should include how many narcotic pills?

A. 0
B. 10
C. 20
D. 30

Correct answer: B

 Discussion

After the designation of pain as the fifth vital sign, opioid analgesic use has steadily increased. Many
surgeons routinely prescribe 30 or more pills after elective hand surgery. However, studies show that
patients generally use fewer than 30 pills. Patients who underwent bone procedures used 14 pills, and
those undergoing soft-tissue procedures used 9 pills. Education and decision aids may help physicians
size prescriptions appropriately to avoid overmedication. Patients undergoing small soft-tissue surgeries
such as trigger releases should not need narcotics. Those undergoing small-joint surgeries, carpal tunnel
releases, and Dupuytren fasciectomy may benefit from a prescription of 10 pills. More extensive
surgery, such as open fracture treatment, may justify more pills, but prescriptions should not exceed 40
tablets under typical circumstances.

 Recommended Readings

Stanek JJ, Renslow MA, Kalliainen LK. The effect of an educational program on opioid prescription
patterns in hand surgery: a quality improvement program. J Hand Surg Am. 2015 Feb;40(2):341-6. doi:
10.1016/j.jhsa.2014.10.054. Epub 2014 Dec 24. PubMed

Rodgers J, Cunningham K, Fitzgerald K, Finnerty E. Opioid consumption following outpatient upper


extremity surgery. J Hand Surg Am. 2012 Apr;37(4):645-50. doi: 10.1016/j.jhsa.2012.01.035. Epub
2012 Mar 10. PubMed
Question 63 of 100

Figures 1 and 2 show the postreduction radiographs obtained from a 32-year-old man who fell from a ladder
onto his outstretched right arm. He reports right wrist pain and dense numbness in his radial digits. What is the
most appropriate treatment option?
Figure 1 Figure 2
A. Emergent surgery, including open carpal tunnel release, open reduction of the perilunate dislocation,
repair of the scapholunate ligament, and intercarpal pinning
B. Emergent surgery, including open carpal tunnel release, closed reduction of the perilunate dislocation,
and casting
C. Elective outpatient surgery, including open carpal tunnel release, open reduction of the perilunate
dislocation, repair of the scapholunate ligament, and intercarpal pinning
D. Emergent surgery, including open reduction of the perilunate dislocation, repair of the scapholunate
ligament, and intercarpal pinning

Correct answer: A

 Discussion

This patient sustained a lesser-arc perilunate dislocation. As a result of the injury, he also developed
acute carpal tunnel syndrome. The closed reduction attempt was unsuccessful; therefore, this injury is
best managed with emergent surgery, an open carpal tunnel release, an open reduction of the perilunate
dislocation, scapholunate ligament repair, and intercarpal pinning. Outpatient surgery in a delayed
fashion is not advised because of the acuity and severity of the carpal tunnel syndrome. Closed reduction
and casting is not advised, because it commonly leads to continued carpal instability with subsequent
dorsal intercalated segment instability deformity and scaphoid lunate advanced collapse wrist arthritis.

 Recommended Readings

Muppavarapu RC, Capo JT. Perilunate Dislocations and Fracture Dislocations. Hand Clin. 2015
Aug;31(3):399-408. doi: 10.1016/j.hcl.2015.04.002. PubMed

Budoff JE. Treatment of acute lunate and perilunate dislocations. J Hand Surg Am. 2008
Oct;33(8):1424-32. doi: 10.1016/j.jhsa.2008.07.016. PubMed

Question 64 of 100

While attempting to recreate the inclination of the distal radius during volar fixation of an intra-articular sagittal
split fracture, use of intraoperative fluoroscopic imaging in the position shown in Figure 1 would be helpful in
showing
Figure 1

A. intra-articular screw penetration.


B. alignment of the joint surface.
C. alignment of the sigmoid notch.
D. dorsal screw penetration.

Correct answer: A

 Discussion

The image demonstrates a rotational fluoroscopic view of the lateral distal radius while attempting to
recreate the inclination of the distal radius. This view is most useful to ensure against intra-articular
screw penetration. The overall alignment of the joint surface is best viewed with a posteroanterior tilt of
11 degrees. The alignment of the sigmoid notch is not seen well on lateral images. Carpal alignment is
seen well on lateral images. Dorsal screw penetration is best viewed dynamically with a flexed wrist
tangential view.

 Recommended Readings

Patel A, Culbertson MD, Lahey P, Semenovski M, Choueka J. Improving accuracy and confidence in
distal radius volar plate screw placement through supplemental radiography: examining specialty,
education, and experience levels. Hand (N Y). 2013 Sep;8(3):308-14. doi: 10.1007/s11552-013-9528-3.
PubMed

Tweet ML, Calfee RP, Stern PJ. Rotational fluoroscopy assists in detection of intra-articular screw
penetration during volar plating of the distal radius. J Hand Surg Am. 2010 Apr;35(4):619-27. doi:
10.1016/j.jhsa.2009.12.033. Epub 2010 Mar 3. PubMed

Question 65 of 100

Figures 1 and 2 show the radiographs obtained from a 56-year-old man who has been experiencing progressive
wrist pain since he felt a pop while throwing a 25-pound bag over his shoulder 6 months ago. Failure to address
the injury surgically might lead to progressive arthritic changes in what order?
Figure 1 Figure 2

A. Lunocapitate, radioscaphoid, radial styloid, radiolunate


B. Radioscaphoid, radial styloid, lunocapitate, radiolunate
C. Radial styloid, radioscaphoid, lunocapitate, radiolunate
D. Radial styloid, radioscaphoid, radiolunate, lunocapitate

Correct answer: C

 Discussion

This patient demonstrates scapholunate dissociation with an associated dorsal intercalated segment
instability deformity. Chronic scapholunate ligament tears lead to scapholunate advanced collapse
(SLAC) wrist. Watson and Ballet describe SLAC wrist as having a predictable progression of arthritic
changes, starting at the radial styloid, progressing to the radioscaphoid joint, and advancing to the
lunocapitate joint. Some authors have described the radiolunate joint as being affected in very late-stage
SLAC wrist.

 Recommended Readings

Watson HK, Ballet FL. The SLAC wrist: scapholunate advanced collapse pattern of degenerative
arthritis. J Hand Surg Am. 1984 May;9(3):358-65. PubMed

Strauch RJ. Scapholunate advanced collapse and scaphoid nonunion advanced collapse arthritis--update
on evaluation and treatment. J Hand Surg Am. 2011 Apr;36(4):729-35. doi: 10.1016/j.jhsa.2011.01.018.
PubMed

Trehan SK, SK Lee, Wolfe SW. Scapholunate Advanced Collapse: Nomenclature and Differential
Diagnosis. J Hand Surg Am. 2015 Oct; 40 (10): 2085-9. doi: 10.1016 / j.jhsa.2015.06.110. Epub 2015
Aug 1. PubMed

Question 66 of 100

Figures 1 through 3 show the radiographs obtained from a 40-year-old woman who injured her right index finger
in a bicycle collision. Failure to restore sagittal plane alignment would likely result in
Figure 1 Figure 2 Figure 3

A. overlapping of the digits.


B. index finger proximal interphalangeal (PIP) joint arthritis.
C. extensor lag at the PIP joint.
D. hyperextension at the PIP joint.

Correct answer: C

 Discussion

The radiographs reveal an extra-articular proximal phalanx fracture of the index finger. The fracture is
comminuted with dorsal angulation of the distal fragment. The question specifically asks about the
restoration of sagittal alignment. The fracture is comminuted with dorsal angulation of the distal
fragment. The other options are incorrect, because overlapping of the digits occurs with rotational
malalignment, the development of arthritis may occur with intra-articular fractures, and hyperextension
would not occur with this type of deformity.
 Recommended Readings

Vahey JW, Wegner DA, Hastings H 3rd. Effect of proximal phalangeal fracture deformity on extensor
tendon function. J Hand Surg Am. 1998 Jul;23(4):673-81. PubMed

Meals C, Meals R. Hand fractures: a review of current treatment strategies. J Hand Surg Am. 2013
May;38(5):1021-31; quiz 1031. doi: 10.1016/j.jhsa.2013.02.017. PubMed

Kozin SH, Thoder JJ, Lieberman G. Operative treatment of metacarpal and phalangeal shaft fractures. J
Am Acad Orthop Surg. 2000 Mar-Apr;8(2):111-21. Full text

Question 67 of 100

Figure 1 depicts an intraoperative photograph obtained following proximal row carpectomy. The black dot
denotes the capitate. The top of the figure is radial and the bottom of the figure is ulnar. Surgical disruption of
the structure identified by the forceps would result in
Figure 1

A. loss of active thumb interphalangeal (IP) flexion.


B. distal radioulnar joint instability.
C. avascular necrosis of the capitate.
D. ulnar carpal translocation.
Correct answer: D

 Discussion

The structure identified by the forceps is the radioscaphocapitate ligament. During a proximal row
carpectomy, it is very important to identify and protect this ligament. Compromise of the ligament
would result in ulnar translocation of the carpus and early failure of the proximal row carpectomy
procedure. If the ligament is injured during surgery, immediate repair should be performed. Green and
associates discuss the importance of the radioscaphocapitate ligament in stabilizing the carpus after this
procedure is performed.

Nakamura and associates compared 3-mm, 6-mm, and 10-mm radial styloidectomies, and only the 3-
mm styloidectomy subsequently preserved carpal stability. Compromise of the radioscaphocapitate
ligament occurred when larger portions of the radial styloid were excised.

Distal radioulnar joint instability would result only from the disruption of the distal radioulnar joint
stabilizers. Avascular necrosis would not occur, because the capitate receives its blood supply mainly
from the palmar vessels. Finally, loss of active thumb IP flexion would not occur, because the flexor
pollicis longus tendon would remain intact even if ligament compromise were to occur.

 Recommended Readings

Nakamura T, Cooney WP 3rd, Lui WH, Haugstvedt JR, Zhao KD, Berglund L, An KN. Radial
styloidectomy: a biomechanical study on stability of the wrist joint. J Hand Surg Am. 2001
Jan;26(1):85-93. PubMed

Green DP, Perreira AC, Longhofer LK. Proximal Row Carpectomy. J Hand Surg Am. 2015
Aug;40(8):1672-6. doi: 10.1016/j.jhsa.2015.04.033. Epub 2015 Jun 25. PubMed

Question 68 of 100

A 23-year-old man cut the dorsal and ulnar aspects of his long finger on a table saw. The dorsal and ulnar skin
over the middle phalanx is missing, with a 2-cm x 2-cm area of loss. There is a 50% loss of the extensor tendon
(ulnar), and the remaining tendon has no tenosynovium. The physician should recommend irrigation and
debridement and

A. tendon repair, and thenar flap coverage.


B. full-thickness skin graft.
C. reversed cross-finger flap from the ring finger.
D. cross-finger flap coverage from the ring finger.

Correct answer: C

 Discussion

The patient has exposed bone and tendon and a partial tendon injury. The remaining radial tendon is
satisfactory and no tendon repair is required. The exposed bone and tendon necessitate vascularized
tissue coverage. A reversed cross-finger flap from the ring finger is suitable for coverage of the dorsal
surface of an adjacent digit.

 Recommended Readings

Kappel DA, Burech JG. The cross-finger flap. An established reconstructive procedure. Hand Clin.
1985 Nov;1(4):677-83. PubMed

Atasoy E. Reversed cross-finger subcutaneous flap. J Hand Surg Am. 1982 Sep;7(5):481-3. PubMed

Question 69 of 100

A 35-year-old man sustained a traumatic low ulnar nerve palsy 18 months ago. The extent of the clawing and
intrinsic atrophy as well as the active radial deviation are seen in Figures 1 through 3. No hyperextensibility of
any of the proximal interphalangeal (PIP) joints is observed. Preoperatively, the patient is not able to fully
extend the PIP joints with the wrist in neutral position and the examiner holding the metacarpophalangeal
(MCP) joints flexed. Figure 4 shows the intraoperative photograph obtained during the intrinsic reconstruction
procedure that is performed. The tendon grafts were inserted distally into the
Figure 1 Figure 2

Figure 3 Figure 4

A. proximal phalanx.
B. radial lateral bands.
C. first annular pulley.
D. second annular pulley.

Correct answer: B

 Discussion
Originally, Burkhalter and Strait recommended bony insertion into the proximal phalanx through a drill
hole. This procedure does require more surgical dissection and flexes only the MCP joints; thus it cannot
extend the PIP joints directly. It does improve clawing in the fingers if the PIPs can extend with
preoperative MCP flexion. The ability to extend the PIP joints is evaluated preoperatively using the
Bouvier test. With the wrist in neutral position, the examiner holds the MCPs flexed and looks for the
ability in that position to actively extend the PIPs. If the patient is able to do so, then the test is
considered positive, and this describes "simple" clawing. In such cases, procedures that flex only the
MCPs are appropriate. The insertion sites for these procedures include the proximal phalanx, the first
annular pulley, and the second annular pulley.

If the Bouvier test is negative, then it is best to insert the tendon grafts distally into the lateral bands.
This technique has a low chance of leading to hyperextension of the PIP joints, particularly when
performed with a wrist extensor motor (which leaves the flexor digitorum superficialis undisturbed) and
with no preoperative hyperextensibility of the PIPs.

 Recommended Readings

Sammer DM, Chung KC. Tendon transfers: Part II. Transfers for ulnar nerve palsy and median nerve
palsy. Plast Reconstr Surg. 2009 Sep;124(3):212e-21e. doi: 10.1097/PRS.0b013e3181b037c7. PubMed

Burkhalter WE, Strait JL. Metacarpophalangeal flexor replacement for intrinsic-muscle paralysis. J
Bone Joint Surg Am. 1973 Dec;55(8):1667-76. PubMed

Muzykewicz DA, Arnet U, Lieber RL, Fridén J. Intrinsic hand muscle function, part 2: kinematic
comparison of 2 reconstructive procedures. J Hand Surg Am. 2013 Nov;38(11):2100-2105.e1. doi:
10.1016/j.jhsa.2013.08.098. PubMed

Question 70 of 100

Figure 1 points to the "tear drop" of the wrist. This radiographic landmark represents which anatomic portion of
the wrist?
Figure 1

A. Ulnar head
B. Volar ulnar corner
C. Radial styloid
D. Lister tubercle

Correct answer: B

 Discussion
Medoff described the radiographic teardrop of the distal radius. This radiographic landmark matches the
critical volar ulnar corner of the distal radius. A malreduction of the volar ulnar corner of the distal
radius in an intra-articular distal radius fracture leads to volar subluxation of the lunate and the rapid
development of posttraumatic arthritis at the distal radioulnar and radiolunate joints. Knowledge of the
specific shape and appearance of this radiographic landmark helps the surgeon when he or she is
critically analyzing postreduction imaging.

The volar portion of the ulnar head may be mistaken for this teardrop sign and should be separately
identified as distinct from the distal radius. The radial styloid and Lister tubercle are not part of the volar
aspect of the lunate facet.

 Recommended Readings

Harness NG, Jupiter JB, Orbay JL, Raskin KB, Fernandez DL. Loss of fixation of the volar lunate facet
fragment in fractures of the distal part of the radius. J Bone Joint Surg Am. 2004 Sep;86-A(9):1900-8.
PubMed

Medoff RJ. Essential radiographic evaluation for distal radius fractures. Hand Clin. 2005
Aug;21(3):279-88. Review. PubMed

Question 71 of 100

Figures 1 through 3 demonstrate the radiographs obtained from a 45-year-old construction worker who has
wrist pain, loss of motion, and loss of strength. Nonsurgical measures have failed, and the patient requests
surgery. What is the best surgical option for this patient?
Figure 1 Figure 2 Figure 3

A. Proximal row carpectomy


B. Excision of the proximal pole of the scaphoid
C. Bone grafting with fixation of the scaphoid
D. Scaphoid excision with four-corner fusion

Correct answer: D

 Discussion

This patient has a physically demanding occupation. His radiographs show a long-standing nonunion
with avascular necrosis and collapse of the proximal pole. This condition is referred to as a scaphoid
nonunion advanced collapse wrist (SNAC). He has advanced arthritis of the capitolunate joint, but the
radiolunate joint is preserved. The best option for him would be scaphoid excision with four-corner
fusion.

Proximal row carpectomy, would be incorrect because of the advanced arthritis of the capitolunate joint.
This procedure relies on normal cartilage/joint articulation of the capitate as well as of the lunate fossa
of the distal radius. Proximal pole scaphoid excision alone would not address the advanced arthritis of
the capitolunate joint. Scaphoid fixation with bone grafting would not address the midcarpal arthritis and
advanced proximal pole collapse.

 Recommended Readings

Shah CM, Stern PJ. Scapholunate advanced collapse (SLAC) and scaphoid nonunion advanced collapse
(SNAC) wrist arthritis. Curr Rev Musculoskelet Med. 2013 Mar;6(1):9-17. doi: 10.1007/s12178-012-
9149-4. PubMed

Weiss KE, Rodner CM. Osteoarthritis of the wrist. J Hand Surg Am. 2007 May-Jun;32(5):725-46.
PubMed

Question 72 of 100

A 54-year-old laborer has a 6-month history of lateral elbow pain. An elbow examination reveals full range of
motion, tenderness over the lateral epicondyle, and pain with resisted wrist extension with the elbow in
extension. Elbow radiograph findings are normal. You perform a steroid injection and the patient's symptoms
are decreased 6 weeks later. One year after receiving the injection, this patient—when compared to a patient
who did not have a steroid injection—is likely to

A. have no difference in elbow pain.


B. no longer have elbow pain.
C. need surgery.
D. experience tendon rupture.

Correct answer: A

 Discussion

This patient has signs and symptoms of lateral epicondylitis. Treatments include various forms of
physical therapy, iontophoresis, corticosteroid injection, nitroglycerin patch treatment, blood injections,
prolotherapy, and surgical intervention. No single treatment is superior to other treatments for this
common problem. Several studies have demonstrated a short-term decrease in symptoms following
steroid injection (6 weeks) but an increased likelihood of persistent symptoms 1 year after treatment.
Steroid injection at this site has not been associated with increased risk for tendon rupture or need for
surgical intervention.

 Recommended Readings

Dong W, Goost H, Lin XB, Burger C, Paul C, Wang ZL, Kong FL, Welle K, Jiang ZC, Kabir K.
Injection therapies for lateral epicondylalgia: a systematic review and Bayesian network meta-analysis.
Br J Sports Med. 2016 Aug;50(15):900-8. doi: 10.1136/bjsports-2014-094387. Epub 2015 Sep 21.
PubMed

Coombes BK, Bisset L, Brooks P, Khan A, Vicenzino B. Effect of corticosteroid injection,


physiotherapy, or both on clinical outcomes in patients with unilateral lateral epicondylalgia: a
randomized controlled trial. JAMA. 2013 Feb 6;309(5):461-9. doi: 10.1001/jama.2013.129. PubMed

Question 73 of 100

A 17-year-old boy with left spastic hemiplegia secondary to cerebral palsy is being evaluated for persistent swan
neck deformities of the affected hand. Splinting has been tried with some improvement, but the patient does
not want to wear the splints any more. On physical examination, he demonstrates full extension of the
metacarpophalangeal (MCP) joints, 30° of hyperextension of the proximal interphalangeal (PIP) joints, and
flexion of the distal interphalangeal (DIP) joints when he attempts to actively extend his digits. He is able to
initiate flexion at the PIP joints with his MCP joints held in neutral extension. He has equal PIP flexion when the
MCP joints are extended and flexed. What is the most appropriate surgical treatment to address his swan neck
deformity?

A. Central slip tenotomy


B. Terminal tendon release
C. Dorsal rerouting of the lateral bands
D. Intrinsic lengthening
Correct answer: A

 Discussion

This patient demonstrates full extension of the MCP joints when he actively extends his fingers,
indicative of overpull of the extrinsic finger extensors. This clinical scenario can be corrected by a
central slip tenotomy.

A terminal tendon release is used to address a Boutonnière deformity. The patient does not demonstrate
intrinsic tightness (equal PIP flexion while the MCP flexed and extended), therefore his swan neck
would be unlikely to respond to intrinsic lengthening. Dorsal rerouting of the lateral bands is performed
for a Boutonnière deformity. A central slip tenotomy would balance the extension forces between the
PIP and DIP joints.

 Recommended Readings

Van Heest AE, House JH. Lateral band rerouting in the treatment of swan neck deformities due to
cerebral palsy. Tech Hand Up Extrem Surg. 1997 Sep;1(3):189-94. PubMed

Carlson EJ, Carlson MG. Treatment of swan neck deformity in cerebral palsy. J Hand Surg Am. 2014
Apr;39(4):768-72. doi: 10.1016/j.jhsa.2014.01.039. Epub 2014 Mar 6. PubMed

Question 74 of 100

You are counseling a 55-year-old woman for a right carpal tunnel release. What can you tell her about the
treatment benefit (grip strength and paresthesia relief) 1 year after surgery compared with continued splinting,
NSAID use, physical therapy, and a single steroid injection?

A. No change in paresthesias and grip strength


B. Increase in grip strength and decrease in paresthesias
C. Decrease in grip strength and increase in paresthesias
D. Increase in grip strength and paresthesias
Correct answer: B

 Discussion

Gerritsen and associates, Hui and associates, and Jarvik and associates compared the effectiveness of
surgical versus nonsurgical treatment for the relief of carpal tunnel symptoms. All three studies showed
that surgery was superior for the relief of paresthesias and the improvement of grip strength. According
to the American Academy of Orthopaedic Surgeons Clinical Guidelines on the Treatment of Carpal
Tunnel Syndrome, strong evidence supports the assertion that surgical treatment of carpal tunnel
syndrome should have a greater treatment benefit at 6 and 12 months than splinting, NSAIDs, physical
therapy, and a single steroid injection.

The other choices, including no change in grip strength and paresthesias, decrease in grip strength and
increase in paresthesias, and increase in grip strength and paresthesias, are not supported by the
evidence.

 Recommended Readings

Hui AC, Wong S, Leung CH, Tong P, Mok V, Poon D, Li-Tsang CW, Wong LK, Boet R. A randomized
controlled trial of surgery vs steroid injection for carpal tunnel syndrome. Neurology. 2005 Jun
28;64(12):2074-8. PubMed

Jarvik JG, Comstock BA, Kliot M, Turner JA, Chan L, Heagerty PJ, Hollingworth W, Kerrigan CL,
Deyo RA. Surgery versus non-surgical therapy for carpal tunnel syndrome: a randomised parallel-group
trial. Lancet. 2009 Sep 26;374(9695):1074-81. doi: 10.1016/S0140-6736(09)61517-8. PubMed

Gerritsen AA, de Vet HC, Scholten RJ, Bertelsmann FW, de Krom MC, Bouter LM. Splinting vs
surgery in the treatment of carpal tunnel syndrome: a randomized controlled trial. JAMA. 2002 Sep
11;288(10):1245-51. PubMed

Graham B, Peljovich AE, Afra R, Cho MS, Gray R, Stephenson J, Gurman A, MacDermid J, Mlady G,
Patel AT, Rempel D, Rozental TD, Salajegheh MK, Keith MW, Jevsevar DS, Shea KG, Bozic KJ,
Adams J, Evans JM, Lubahn J, Ray WZ, Spinner R, Thomson G, Shaffer WO, Cummins DS, Murray
JN, Mohiuddin M, Mullen K, Shores P, Woznica A, Linskey E, Martinez Y, Sevarino K. The American
Academy of Orthopaedic Surgeons Evidence-Based Clinical Practice Guideline on: Management of
Carpal Tunnel Syndrome. J Bone Joint Surg Am. 2016 Oct 19;98(20):1750-1754. PubMed

Question 75 of 100
Figures 1 and 2 display the radiographs obtained from a woman who had volar plating of the distal radius 8
months earlier. Two days ago, she noticed she could not actively extend her thumb. What is the most
appropriate treatment that would restore active thumb extension?

Figure 1 Figure 2

A. Removal of hardware with tendon transfer


B. Repair of the extensor pollicis longus (EPL) tendon primarily
C. Thumb interphalangeal (IP) arthrodesis
D. Nonsurgical treatment with cast placement keeping the thumb in a fully extended position for 4 weeks

Correct answer: A

 Discussion

Although the fracture is aligned in anatomic position, prominence of a least one of the distal screws is
evident on the lateral radiographic view. The prominent screw is the most likely cause of the EPL
tendon rupture. If the patient chooses surgical treatment, the best option would be removal of the
offending hardware combined with extensor indicis proprius to EPL tendon transfer. Intercalary grafting
would also be an acceptable option. If the tendon transfer were to be performed alone, the prominent
screw(s) could rupture the transferred tendon as well. Also, it is rarely possible to repair the EPL tendon
primarily in such cases, because this rupture is an attrition type. Casting would obviously not provide
any benefit in this situation, and IP arthrodesis would not be the first surgical treatment option.

This problem can be avoided by using shorter screws or not placing screws in plate holes that direct
screws into the third dorsal extensor compartment. Intraoperative fluoroscopy and special views, such as
the carpal shoot-through view, are useful for avoiding this complication.

 Recommended Readings

Benson EC, DeCarvalho A, Mikola EA, Veitch JM, Moneim MS. Two potential causes of EPL rupture
after distal radius volar plate fixation. Clin Orthop Relat Res. 2006 Oct;451:218-22. PubMed

Berglund LM, Messer TM. Complications of volar plate fixation for managing distal radius fractures. J
Am Acad Orthop Surg. 2009 Jun;17(6):369-77. Full text

Question 76 of 100

When performing a radioscapholunate (RSL) fusion for posttraumatic radiocarpal arthritis, excision of the distal
pole of the scaphoid will cause a decrease in

A. the nonunion rate.


B. wrist extension.
C. carpal height.
D. avascular necrosis.

Correct answer: A

 Discussion

RSL arthrodesis is a motion-sparing option for posttraumatic radioscaphoid or radiolunate arthritis when
the midcarpal joint is preserved. Preserving the midcarpal joint allows the dart-thrower motion to
remain. Mühldorfer-Fodor and associates reported that the rates of nonunion for RSL fusion were
reduced by excision of the distal pole of the scaphoid. Multiple studies have shown increased radial and
ulnar deviation with excision of the distal pole of the scaphoid; excision of the triquetrum further
increases the radial-ulnar arc of motion. Bain and associates and Pervaiz and associates reported
increased wrist flexion-extension arcs with distal scaphoid and triquetrum excisions in cadaveric
models; other authors have reported no difference.

 Recommended Readings

Bain GI, Sood A, Yeo CJ. RSL Fusion with Excision of Distal Scaphoid and Triquetrum: A Cadaveric
Study. J Wrist Surg. 2014 Feb;3(1):37-41. doi: 10.1055/s-0033-1364095. PubMed

Mühldorfer-Fodor M, Ha HP, Hohendorff B, Löw S, Prommersberger KJ, van Schoonhoven J. Results


after radioscapholunate arthrodesis with or without resection of the distal scaphoid pole. J Hand Surg
Am. 2012 Nov;37(11):2233-9. doi: 10.1016/j.jhsa.2012.08.009. PubMed

Pervaiz K, Bowers WH, Isaacs JE, Owen JR, Wayne JS. Range of motion effects of distal pole scaphoid
excision and triquetral excision after radioscapholunate fusion: a cadaver study. J Hand Surg Am. 2009
May-Jun;34(5):832-7. doi: 10.1016/j.jhsa.2009.02.007. PubMed

Question 77 of 100

Figures 1 and 2 are the radiographs of a 17-year-old man who injured his wrist 6 months ago. He is experiencing
pain and limited motion. What is the most effective treatment option?
Figure 1 Figure 2

A. Long-arm thumb spica casting


B. Bracing and bone stimulation
C. Scaphoid excision with intercarpal fusion
D. Bone grafting with screw placement

Correct answer: D
 Discussion

Figures 1 and 2 show a scaphoid nonunion with substantial bone resorption at the nonunion site. Cast
immobilization and bracing with bone stimulator use would not be successful treatments at this point
because the fracture is 6 months old and there is considerable bone resorption at the fracture site.
Scaphoid excision with intercarpal fusion is an option to use only after bone-grafting procedures have
failed or arthritis is present.

Bone-grafting procedures using both vascularized and nonvascularized graft sources are associated with
a high success rate that decreases with avascular necrosis of the proximal pole. If left untreated,
scaphoid nonunions can progress to carpal collapse and degenerative arthritis.

 Recommended Readings

Buijze GA, Ochtman L, Ring D. Management of scaphoid nonunion. J Hand Surg Am. 2012
May;37(5):1095-100; quiz 1101. doi: 10.1016/j.jhsa.2012.03.002. Review. PubMed

Kawamura K, Chung KC. Treatment of scaphoid fractures and nonunions. J Hand Surg Am. 2008 Jul-
Aug;33(6):988-97. doi: 10.1016/j.jhsa.2008.04.026. Review. PubMed

Question 78 of 100

A 32-year-old man sustained an injury to the right thumb metacarpophalangeal (MP) joint ulnar collateral
ligament (UCL) and is undergoing surgical repair (Figure 1). What structure in the clinical photograph is blocking
reduction of the ulnar collateral ligament?
Figure 1

A. Extensor pollicis longus (EPL) tendon


B. Adductor aponeurosis
C. EPB and dorsal capsule
D. Ulnar sesamoid bone and volar plate

Correct answer: B

 Discussion

When the thumb MP UCL is torn from the proximal phalanx, the distal stump can be displaced
superficial to the adductor aponeurosis, known as a Stener lesion. The adductor aponeurosis effectively
blocks reduction of the ligament to the normal attachment site. The EPB and EPL tendons are dorsal to
the UCL, and the ulnar sesamoid bone/volar plate are in a volar position in relation to the UCL. The
dorsal capsule would also not block reduction of the UCL due to it's anatomic location. The other
responses do not block the UCL with this type of injury.
 Recommended Readings

Carlson MG, Warner KK, Meyers KN, Hearns KA, Kok PL. Anatomy of the thumb
metacarpophalangeal ulnar and radial collateral ligaments. J Hand Surg Am. 2012 Oct;37(10):2021-6.
doi: 10.1016/j.jhsa.2012.06.024. Epub 2012 Aug 31. PubMed

Bean CH, Tencer AF, Trumble TE. The effect of thumb metacarpophalangeal ulnar collateral ligament
attachment site on joint range of motion: an in vitro study. J Hand Surg Am. 1999 Mar;24(2):283-7.
PubMed

STENER B. Skeletal injuries associated with rupture of the ulnar collateral ligament of the
metacarpophalangeal joint of the thumb. A clinical and anatomical study. Acta Chir Scand. 1963
Jun;125:583-6. PubMed

Question 79 of 100

At a minimum 2-year follow-up and compared with the metacarpophalangeal (MCP) joint, pyrolytic carbon
resurfacing arthroplasties of the proximal interphalangeal (PIP) joint

A. produce less squeaking or clicking.


B. result in more dislocations.
C. provide superior pain relief.
D. result in better motion compared with the preoperative status.

Correct answer: B

 Discussion

Wall and Stern published a report on MCP joint pyrolytic carbon arthroplasty for osteoarthritis and
another on PIP joint pyrolytic carbon resurfacing arthroplasty for osteoarthritis. They found different
outcomes, and MCP joint implants outperformed PIP joint implants. Of eleven MCP joint arthroplasties,
two produced asymptomatic squeaking and clicking, whereas eleven of 31 PIP joint implants produced
this problem. No dislocations were reported among the MCP joint implants, but five PIP joint
dislocations were observed.

Outcomes were measured by the Michigan Hand Outcomes Questionnaire in both studies and were
satisfactory for the MCP joint implants, with an average score of 80. The PIP implants did not fare as
well, showing a higher degree of pain along with an average score of 53. The authors noted that, in the
15 patients in the PIP study who had unilateral surgery, the uninvolved, nonsurgical hand motion was
actually statistically significantly (P<0.01) better than the surgical hand. MCP joint motion increased
from 62º before surgery to 76º after surgery, whereas PIP joint motion got worse after surgery, with the
average motion decreasing from 57º to 31º.

 Recommended Readings

Wall LB, Stern PJ. Clinical and radiographic outcomes of metacarpophalangeal joint pyrolytic carbon
arthroplasty for osteoarthritis. J Hand Surg Am. 2013 Mar;38(3):537-43. doi:
10.1016/j.jhsa.2012.11.026. Epub 2013 Jan 20. PubMed

Sweets TM, Stern PJ. Pyrolytic carbon resurfacing arthroplasty for osteoarthritis of the proximal
interphalangeal joint of the finger. J Bone Joint Surg Am. 2011 Aug 3;93(15):1417-25. doi:
10.2106/JBJS.J.00832. PubMed

Question 80 of 100

Figure 1 is the right hand of a 65-year-old man with a history of hypertension and rheumatoid arthritis. He is
taking immunosuppressive disease-modifying antirheumatic drugs (DMARDs) and is seen in the emergency
department with rapid progression of erythema from his right thumb to his right arm during the last 12 hours.
He is confused, lethargic, and has these vital signs: blood pressure 92/40, respiratory rate 45, temperature
39.7°C, pulse 135, and oxygen saturation 90% on 4 liters of oxygen by face mask. An examination of his right
upper extremity reveals black bulla extending from the metacarpophalangeal down to the tip and no capillary
refill at the pulp. Immediate treatment should consist of
Figure 1

A. intravenous (IV) antibiotics and admission to a medical intensive care unit.


B. emergent radical debridement including thumb amputation.
C. emergent revascularization of the thumb with a vein graft.
D. urgent irrigation of the thumb flexor tendon sheath.

Correct answer: B

 Discussion

This patient has multiple criteria for necrotizing soft-tissue infection (NSTI, also known as necrotizing
fasciitis) including rapidly progressive infection, black bulla, hypotension and hypoxia, and a history of
immune compromise. Aggressive emergent debridement including the removal of all necrotic tissue and
IV antibiotics can decrease morbidity and mortality.

Not all patients will have such obvious NSTI findings. In less clear cases, a scoring system using
laboratory values (the Laboratory Risk Indicator for Necrotizing Fasciitis) can help clarify the diagnosis.
IV antibiotics are key to treatment as well, but any delay in surgical treatment can increase morbidity
and mortality. The black bulla and necrotic-appearing thumb indicate that this infection is not confined
to the flexor sheath, therefore irrigation of the tendon sheath alone would be insufficient treatment.
Although the thumb is dysvascular, this is because of an infection, and revascularization is not indicated.

 Recommended Readings
Wong CH, Khin LW, Heng KS, Tan KC, Low CO. The LRINEC (Laboratory Risk Indicator for
Necrotizing Fasciitis) score: a tool for distinguishing necrotizing fasciitis from other soft tissue
infections. Crit Care Med. 2004 Jul;32(7):1535-41. PubMed

Gonzalez MH, Kay T, Weinzweig N, Brown A, Pulvirenti J. Necrotizing fasciitis of the upper extremity.
J Hand Surg Am. 1996 Jul;21(4):689-92. PubMed

Question 81 of 100

A 72-year-old woman with diabetes mellitus has right hand numbness. Provocative test findings are consistent
with carpal tunnel syndrome, and electrodiagnostic study (EDS) findings show prolonged median motor and
sensory distal latencies with low-amplitude thenar compound muscle action potential. Poor prognosis is most
associated with which factor?

A. Diabetes
B. Older age
C. Female gender
D. Severity of EDS findings

Correct answer: D

 Discussion

Although carpal tunnel syndrome remains a clinical diagnosis, EDS findings have become important
tools. Moderate disease is defined as abnormal median sensory distal latency and prolonged median
motor distal latency. Severe disease is defined as prolonged median motor and sensory distal latencies
witheither absent sensory nerve action potential or mixed nerve action potential or low-amplitude or
absent thenar compound muscle action potential. Although female gender and diabetes are considered
risk factors, there is little association with postsurgical outcome. Severe disease as confirmed by EDS is
the factor most strongly associated with poor recovery after surgery.

 Recommended Readings
Stevens JC. AAEM minimonograph #26: the electrodiagnosis of carpal tunnel syndrome. American
Association of Electrodiagnostic Medicine. Muscle Nerve. 1997 Dec;20(12):1477-86. PubMed

Tomaino MM, Weiser RW. Carpal tunnel release for advanced disease in patients 70 years and older:
does outcome from the patient’s perspective justify surgery? J Hand Surg Br. 2001 Oct;26(5):481-3.
PubMed

Bland JD. Do nerve conduction studies predict the outcome of carpal tunnel decompression? Muscle
Nerve. 2001 Jul;24(7):935-40. PubMed

Bland JD. Carpal tunnel syndrome. BMJ. 2007 Aug 18;335(7615):343-6. PubMed

Kronlage SC, Menendez ME. The benefit of carpal tunnel release in patients with electrophysiologically
moderate and severe disease. J Hand Surg Am. 2015 Mar;40(3):438-444.e1. doi: 10.1016/j.
jhsa.2014.12.012. PubMed

Question 82 of 100

A 51-year-old male 2-pack per day smoker presents with a hyperkeratotic light brown plaque on the dorsum of
his left ring finger that has been present for 7 years. It measures 14 mm by 13 mm. Initially, it responded to
topical wart treatments, but has failed to do so recently so he sought evaluation by a dermatologist who
biopsied the lesion. The results revealed squamous cell carcinoma (SCC) in situ, and he was referred for further
surgical management. He has no other skin lesions, no history of SCC and no axillary lymphadenopathy. What is
the next step in management?

A. Continued observation and re-evaluation in 6 months


B. Sentinel lymph node biopsy
C. Wide excision
D. Primary ray resection

Correct answer : C

Discussion

SCC in situ is a low-grade malignancy that typically presents as painless lesions on areas of high sun exposure
such as the dorsum of the hand and fingers. The recommended treatment for lesions smaller than 100 mm is
wide excision with 4 mm margins to a depth 1 layer below the tumor, along with any adjacent area of
induration. Sentinel lymph node biopsy is typically not indicated in the setting of a low-grade tumor such as this
one and in the absence of axillary lymphadenopathy.
 Recommended Readings

Askari M, Kakar S, Moran SL. Squamous cell carcinoma of the hand: a 20-year review. J Hand Surg
Am. 2013 Nov;38(11):2124-33. doi: 10.1016/j.jhsa.2013.08.090. Epub 2013 Sep 17. PubMed.

Ilyas EN, Leinberry CF, Ilyas AM. Skin cancers of the hand and upper extremity. J Hand Surg Am.
2012 Jan;37(1):171-8. doi: 10.1016/j.jhsa.2011.10.042. Review. PubMed.

Question 83 of 100

At the first postoperative visit after mini-open carpal tunnel release, a patient reports hand weakness. Poor
index finger interphalangeal joint extension and metacarpophalangeal joint flexion are present. This finding is
most consistent with

A. unrecognized injury to the recurrent motor branch.


B. neuropraxia of the proper palmar digital nerve.
C. new-onset stenosing flexor tenosynovitis.
D. injury to the flexor digitorum profundus to the index finger.

Correct answer: B

 Discussion

Complications after carpal tunnel release are relatively uncommon. The clinical picture described above
is most consistent with lumbrical muscle weakness secondary to neuropraxia of the proper palmar digital
nerve to the index finger supplying motor innervation to that muscle. The recurrent motor branch of the
median nerve innervates the thenar musculature and would not present as index finger weakness. A new
onset of trigger finger may result from a loss of the pulley effect of the transverse carpal ligament,
postoperative tendon inflammation, or previously unrecognized flexor tendon triggering. Flexor
digitorum profundus to the index finger lies deep within the carpal tunnel, making its injury unlikely. If
it were injured, the result would not be weakness of interphalangeal joint extension.
 Recommended Readings

Karl JW, Gancarczyk SM, Strauch RJ. Complications of Carpal Tunnel Release. Orthop Clin North Am.
2016 Apr;47(2):425-33. doi: 10.1016/j.ocl.2015.09.015. Review. PubMed

Seiler JG 3rd, Daruwalla JH, Payne SH, Faucher GK. Normal Palmar Anatomy and Variations That
Impact Median Nerve Decompression. J Am Acad Orthop Surg. 2017 Sep;25(9):e194-e203. Full text

Question 84 of 100

A 35-year-old man who is left-hand dominant has pain and swelling around his left index metacarpal phalangeal
(MCP) joint following a motor vehicle accident 2 months ago. Radiographs reveal no fractures. He has point
tenderness over the radial side of the MCP joint and increased laxity with ulnarly applied stress. He has failed
conservative treatment including 5 weeks of immobilization. If the patient elects to live with this condition and
not have surgery, what would be the most common outcome?

A. Development of a trigger finger


B. Presence of intrinsic tightness
C. Weakness of pinch strength
D. Subluxation of the extensor tendon with MCP joint motion

Correct answer: C

 Discussion

The thumb MCP ulnar collateral ligament (UCL) and index MCP radial collateral ligament (RCL) are
the primary stabilizers to pinch. Secondary stabilizers include the volar plate, dorsal capsule, and
accessory collateral ligaments. With insufficiency of either the thumb MCP UCL or the index MCP
RCL, pinch strength will be severely weakened. The other options listed are unlikely to occur with this
particular injury.
 Recommended Readings

Lourie GM, Gaston RG, Freeland AE. Collateral ligament injuries of the metacarpophalangeal joints of
the fingers. Hand Clin. 2006 Aug;22(3):357-64, viii. PubMed

Dy CJ, Tucker SM, Kok PL, Hearns KA, Carlson MG. Anatomy of the radial collateral ligament of the
index metacarpophalangeal joint. J Hand Surg Am. 2013 Jan;38(1):124-8. doi:
10.1016/j.jhsa.2012.09.032. Epub 2012 Nov 30. PubMed

Gaston RG, Lourie GM. Radial collateral ligament injury of the index metacarpophalangeal joint: an
underreported but important injury. J Hand Surg Am. 2006 Oct;31(8):1355-61. PubMed

Question 85 of 100

Figure 1 shows an injury sustained by a 60-year-old man 4 weeks ago. Since that time he has had substantial
pain and catching of his finger during attempts at range of motion. What is the most appropriate treatment at
this point?

Figure 1
A. Tendon debridement
B. Release of the A2 pulley
C. Tendon repair with core sutures
D. Tendon repair with epitendinous sutures

Correct answer: D

 Discussion

Approximately 70% laceration of the flexor digitorum profundus tendon with active locking is best
treated with epitendinous sutures. Performing this procedure under local anesthetic allows for better
assessment of whether the triggering has been resolved. Cyclic loading has been shown to increase with
high-grade partial lacerations. Use of core sutures adds little strength to a partial laceration. Debridement
alone is reserved for injuries involving less than 60% of the tendon diameter. Release of the A2
wouldcompromise pulley function.

 Recommended Readings

Mostofi A, Palmer J, Akelman E. Flexor tendon injury. In: Chung KC, ed. Hand Surgery Update V.
Rosemont, IL: American Academy of Orthopaedic Surgeons; 2012:181-192.

Haddad R, Scherman P, Peltz T, Nicklin S, Walsh WR. A biomechanical assessment of repair versus
nonrepair of sheep flexor tendons lacerated to 75 percent. J Hand Surg Am. 2010 Apr;35(4):546-51. doi:
10.1016/j.jhsa.2009.12.039. Epub 2010 Mar 2. PubMed

Question 86 of 100

Figures 1 through 3 are the radiographs of a 65-year-old man who sustained a fracture from a fall. The patient
elects open reduction and internal fixation of the distal radius. After plating the distal radius, the distal
radioulnar joint (DRUJ) is examined and found to be unstable in both pronation and supination. What is the best
next step?
Figure 1 Figure 2 Figure 3

A. Early range of motion (ROM) program with a removable short-arm splint


B. Long-arm casting in pronation for 4 weeks
C. Pin fixation of the DRUJ
D. Fixation of the ulnar styloid fracture

Correct answer: D

 Discussion
Figure 4

The initial radiographs show a comminuted displaced distal radius fracture, along with a displaced
fracture of the base of the ulnar styloid. The displacement is best seen on the oblique view. After
reduction and fixation of the radius, DRUJ stability should be assessed. The majority of scenarios that
involve this injury pattern will not be unstable because of the oblique band of the interosseous ligament.
When DRUJ instability is present after fixation of the radius, reduction and fixation of the ulnar styloid
fracture is the best option to provide stability of the distal radioulnar joint (DRUJ) (Figure 4). A study by
Lawton and associates revealed that all distal radius fractures complicated by DRUJ instability were
accompanied by an ulnar styloid fracture. A fracture at the ulnar styloid’s base and substantial
displacement of an ulnar styloid fracture were found to increase risk for DRUJ instability. An ulnar
styloid base fracture involves the insertion of the radioulnar ligaments and can cause DRUJ instability if
displaced. If persistent instability is present after fixation of the ulnar styloid, DRUJ pinning is a
reasonable option. Early ROM with splinting would not allow reduction or healing of the ulnar styloid
and would result in persistent instability. Short-arm casting also would not allow stability of the DRUJ
and would be a less reliable method with which to achieve healing of the ulnar styloid.
 Recommended Readings

Wysocki RW, Ruch DS. Ulnar styloid fracture with distal radius fracture. J Hand Surg Am. 2012
Mar;37(3):568-9. doi: 10.1016/j.jhsa.2011.08.035. Epub 2011 Oct 22. Review. PubMed

Sammer DM, Shah HM, Shauver MJ, Chung KC. The effect of ulnar styloid fractures on patient-rated
outcomes after volar locking plating of distal radius fractures. J Hand Surg Am. 2009 Nov;34(9):1595-
602. doi: 10.1016/j.jhsa.2009.05.017. PubMed

May MM, Lawton JN, Blazar PE. Ulnar styloid fractures associated with distal radius fractures:
incidence and implications for distal radioulnar joint instability. J Hand Surg Am. 2002 Nov;27(6):965-
71. PubMed

Question 87 of 100

A 45-year-old man feels a pop in the anterior aspect of his elbow while lifting furniture. He denies any
antecedent pain or injury. Which examination method is best for diagnosing a distal biceps rupture?

A. The examiner brings a finger from medial to lateral across the antecubital fossa, feeling for a cord-like
structure.
B. The examiner brings a finger from lateral to medial across the antecubital fossa, feeling for a cord-like
structure.
C. With the elbow flexed to 90°and the forearm pronated, the examiner resists patient supination,
evaluating for pain at the bicipital groove.
D. With the patient’s arm elevated to 90° of forward flexion, the elbow extended, and the forearm
supinated, the examiner resists elevation distal to the elbow, evaluating for pain at the bicipital groove.

Correct answer: B

 Discussion

Rupture of the distal biceps tendon is predictably identified by the hook test, which is performed by
bringing a finger from lateral to medial across the antecubital fossa of a flexed elbow, feeling for a cord-
like structure on which the examiner can "hook" a finger. Bringing the finger from medial to lateral can
cause a false-negative result, hooking the lacertus fibrosus, which can remain intact even with a ruptured
distal biceps tendon. The Yergason test (option 3) and the Speed test (option 4) are used to assist in
diagnosing proximal, not distal, biceps and labral pathology. Even if the distal biceps tendon is ruptured,
the supinator remains intact. Although supination weakness may be present, an inability to supinate
should not be observed.

 Recommended Readings

Miyamoto RG, Elser F, Millett PJ. Distal biceps tendon injuries. J Bone Joint Surg Am. 2010 Sep
1;92(11):2128-38. doi: 10.2106/JBJS.I.01213. PubMed

O'Driscoll SW, Goncalves LB, Dietz P. The hook test for distal biceps tendon avulsion. Am J Sports
Med. 2007 Nov;35(11):1865-9. Epub 2007 Aug 8. PubMed

Question 88 of 100

A 75-year-old man has persistent radial sided hand and wrist pain. Radiographs demonstrate severe
scaphotrapezial trapezoidal arthritis. His basal joint is unaffected. His pain has failed to improve with bracing,
activity modification, and image-guided corticosteroid injection. He has elected surgical treatment. What long-
term complication can arise from a distal scaphoid resection?

A. Avascular necrosis of the proximal pole of the scaphoid


B. Dorsal intercalated segment instability (DISI)
C. Volar intercalated segment instability
D. Thumb metacarpophalangeal joint hyperextension

Correct answer: B

 Discussion

Resection of the distal pole of the scaphoid eliminates the arthritic contact at the
scaphotrapeziotrapezoid joint; however, it functionally shortens the scaphoid. Theoretically, the lunate is
at equilibrium between the extension moment of the capitate and the triquetrum and the flexion moment
of the scaphoid. Shortening the scaphoid allows the extension moment of the triquetrum to predominate,
pulling the lunate into extension and creating a DISI deformity. Concomitant capsulodesis or
interposition is recommended by some authors to prevent this complication.

 Recommended Readings

Lluch AL, Garcia-Elias M, Lluch AB. Arthroplasty of the scaphoid-trapezium-trapezoid and


carpometacarpal joints. Hand Clin. 2013 Feb;29(1):57-68. doi: 10.1016/j.hcl.2012.08.021. PubMed

Corbin C, Warwick D. Midcarpal instability after excision arthroplasty for scapho-trapezial-trapezoid


(STT) arthritis. J Hand Surg Eur Vol. 2009 Aug;34(4):537-8. doi: 10.1177/1753193408098903.
PubMed

Question 89 of 100

Figures 1 through 3 are the clinical photograph and radiographs of a 25-year-old, left-hand-dominant man who
injured his left index finger. Which treatment option will most effectively allow satisfactory fracture alignment
and maximize motion?

Figure 1 Figure 2 Figure 3


A. Buddy-taping to the long finger with an early range of motion (ROM) program
B. Closed reduction and static external fixation in extension
C. Open reduction and internal fixation (ORIF) with an early ROM program
D. Digital splinting for 4 weeks followed by a ROM program

Correct answer: C

 Discussion

Figure 4 Figure 5

This patient has an oblique index proximal phalanx fracture with malrotation. Buddy-taping and digital
splinting would not predictably maintain fracture reduction and would result in a malunion with
rotational deformity and possible shortening. Closed reduction and spanning external fixation in
extension would result in significant digital stiffness. ORIF (Figure 4 and 5) followed by an early ROM
program would allow anatomic fracture alignment and give this patient the best chance to regain the
majority of motion in the shortest amount of time.

 Recommended Readings

Kawamura K, Chung KC. Fixation choices for closed simple unstable oblique phalangeal and
metacarpal fractures. Hand Clin. 2006 Aug;22(3):287-95. Review. PubMed

Pun WK, Chow SP, So YC, Luk KD, Ngai WK, Ip FK, Peng WH, Ng C, Crosby C. Unstable phalangeal
fractures: treatment by A.O. screw and plate fixation. J Hand Surg Am. 1991 Jan;16(1):113-7. PubMed

Question 90 of 100

Figures 1 and 2 are the clinical photographs of a 36-year-old woman who cannot fully extend the
metacarpophalangeal (MP) joints of her long and ring fingers 9 months after the removal of a plate from the
proximal radius via a dorsal approach. What is the most likely cause of this problem?

Figure 1 Figure 2

A. Laceration of a branch of the posterior interosseous nerve (PIN)


B. Postsurgical tendon adhesion
C. Laceration of the (EDC) tendons to long and ring fingers
D. Neuropraxia of the PIN

Correct answer: A

 Discussion

Plating of the proximal radius from a dorsal or Thompson approach (between the extensor carpi radialis
brevis and extensor digitorum communis) allows complete visualization of the PIN through the
supinator. Going through the same incision from proximal to distal produces a scar-filled proximal
approach, and it is not uncommon to drift ulnarly and injure an individual nerve to the extensor
digitorum communis muscle, resulting in the deformity seen in Figure 1 and 2.

Postsurgical tendon adhesions rarely involve only 2 tendons. Complete tendon lacerations are rare at this
level in the proximal forearm, as is a complete laceration of the PIN. Neuropraxias are common but
usually resolve after 3 months. The gossamer-thin branch of the PIN to the EDC can be easily damaged
in scar tissue, resulting in an inability to fully extend the MP joints of the long and ring fingers. The
proprius tendons allow the patient to fully extend the index and little finger MP joints. Connecting the
EDC of the long finger to the extensor indicis proprius and the EDC of the ring finger to the extensor
digiti quinti proprius can correct the deformity. To avoid the problem, the surgeon should start the
incision distally in normal anatomy, and the interval between the mobile wad and the digital extensors
will be more easily found.

 Recommended Readings

Dang AC, Rodner CM. Unusual compression neuropathies of the forearm, part I: radial nerve. J Hand
Surg Am. 2009 Dec;34(10):1906-14. doi: 10.1016/j.jhsa.2009.10.016. PubMed

Konjengbam M, Elangbam J. Radial nerve in the radial tunnel: anatomic sites of entrapment neuropathy.
Clin Anat. 2004 Jan;17(1):21-5. PubMed

Lister GD, Belsole RB, Kleinert HE. The radial tunnel syndrome. J Hand Surg Am. 1979 Jan;4(1):52-9.
PubMed
Question 91 of 100

Which characteristic of complex regional pain syndrome (CRPS) type 2 differentiates it from CRPS 1?

A. Positive bone scan result


B. Identified nerve injury
C. Pseudomotor changes
D. No identified nerve injury

Correct answer: B

 Discussion

CRPS 2 develops after nerve injury, whereas CRPS 1 occurs without nerve injury. The diagnostic
criteria are otherwise the same for the 2 conditions. A 3-phase bone scan can be helpful; a pattern of
increased uptake in all 3 phases, and particularly diffuse periarticular uptake in and around the joints of
the affected extremity during the delayed phase, is considered typical of CRPS, especially during the
first 6 months; however, it is not specific enough to be used as a diagnostic criteria.

Unfortunately, the pathophysiological mechanisms underlying CRPS remain unclear, and, as such, no
standard diagnostic test for CRPS exists. There are 3 major sets of diagnostic criteria for CRPS: the
International Association for the Study of Pain (IASP) criteria, the Budapest Research Criteria, and the
Veldman criteria. According to IASP, CRPS type 1 involves an initiating noxious event or a cause of
immobilization; continuing pain/allodynia/hyperalgesia for which the pain is disproportionate to any
known inciting event; evidence of edema, changes in skin blood flow, or abnormal pseudomotor activity
in the region of pain; and exclusion of the diagnosis by the existence of other conditions that would
otherwise account for the degree of pain and dysfunction. CRPS type 2 is a syndrome that develops after
nerve injury, contends IASP. Spontaneous pain or allodynia/hyperalgesia occurs and is not necessarily
limited to the territory of the injured nerve; there is or has been evidence of edema, a skin blood flow
abnormality, or abnormal sudomotor activity in the region of pain since the inciting event; and the
diagnosis is excluded by the existence of conditions that would otherwise account for the degree of pain
and dysfunction.

 Recommended Readings
Patterson RW, Li Z, Smith BP, Smith TL, Koman LA. Complex regional pain syndrome of the upper
extremity. J Hand Surg Am. 2011 Sep;36(9):1553-62. doi: 10.1016/j.jhsa.2011.06.027. Review. PubMed

Cappello ZJ, Kasdan ML, Louis DS. Meta-analysis of imaging techniques for the diagnosis of complex
regional pain syndrome type I. J Hand Surg Am. 2012 Feb;37(2):288-96. doi:
10.1016/j.jhsa.2011.10.035. Epub 2011 Dec 15. Review. PubMed

Patterson RW, Li Z, Smith BP, Smith TL, Koman LA. Complex regional pain syndrome of the upper
extremity. J Hand Surg Am. 2011 Sep;36(9):1553-62. doi: 10.1016/j.jhsa.2011.06.027. Review. PubMed

Borchers AT, Gershwin ME. Complex regional pain syndrome: a comprehensive and critical review.
Autoimmun Rev. 2014 Mar;13(3):242-65. doi: 10.1016/j.autrev.2013.10.006. Epub 2013 Oct 23.
Review. PubMed

Question 92 of 100

A 35-year-old man has a brachial plexus injury affecting the lateral cord. He partially improves with observation
and now has complete return of median nerve function and pectoral muscle function. What nerve transfer is
most likely to restore the motor function he is lacking?

A. Median and ulnar fascicles to musculocutaneous nerve transfer


B. Medial triceps branch to axillary nerve transfer
C. Intercostal nerve to triceps branch of radial nerve transfer
D. Anterior interosseous nerve (AIN) to ulnar motor transfer

Correct answer: A

 Discussion

The lateral cord of the brachial plexus gives off the lateral pectoral nerve, the musculocutaneous nerve,
and then contributes to the median nerve. The patient has had recovery of function of these components
except for the musculocutaneous nerve. The musculocutaneous nerve innervates the biceps and the
brachialis, which provide elbow flexion. To restore motor function, a nerve transfer would have to
provide reinnervation of the biceps and brachialis.
 Recommended Readings

Ray WZ, Pet MA, Yee A, Mackinnon SE. Double fascicular nerve transfer to the biceps and brachialis
muscles after brachial plexus injury: clinical outcomes in a series of 29 cases. J Neurosurg. 2011
Jun;114(6):1520-8. doi: 10.3171/2011.1.JNS10810. Epub 2011 Feb 25. PubMed

Tung TH, Mackinnon SE. Nerve transfers: indications, techniques, and outcomes. J Hand Surg Am.
2010 Feb;35(2):332-41. doi: 10.1016/j.jhsa.2009.12.002. PubMed

Question 93 of 100

Figures 1 and 2 show the MRI studies obtained from a 35-year-old manual laborer with persistent wrist pain
despite immobilization. At the time of surgery, collapse of the capitate and arthritic changes of the midcarpal
joint are noted. What is the most appropriate procedure for this condition?

Figure 1 Figure 2

A. Local vascularized bone graft


B. Proximal row carpectomy
C. Midcarpal fusion
D. Total wrist arthroplasty

Correct answer: C

 Discussion

The T1-weighted MRI reveals decreased signal that is consistent with avascular necrosis (AVN) of the
capitate. Figure 2 demonstrates increased signal of the capitate consistent with edema. The etiology of
AVN of the capitate may be related to trauma, abnormal interosseous vascular supply, and
hypermobility. Surgical treatment is considered for patients who have had persistent symptoms despite
immobilization. At the time of surgery, collapse of the capitate and arthritic changes would be treated
most appropriately with a salvage procedure. A midcarpal fusion is a motion-preserving salvage
procedure and is the most appropriate option given to address the pain associated with the midcarpal
arthritic changes. The alternative options are not appropriate for this patient. Local vascularized bone
grafts are considered for situations in which no evidence of capitate collapse or arthritis is observed. A
proximal row carpectomy would lead to suboptimal results, because the collapsed proximal pole of the
capitate would articulate with the lunate facet of the radius. In addition, a high rate of failure has been
seen in young patients requiring full wrist fusion following proximal row carpectomy. A total wrist
arthroplasty is a salvage procedure but is not indicated in this young manual laborer. This patient
population experiences a high rate of failure due to mechanical prosthetic complications.

 Recommended Readings

Hattori Y, Doi K, Sakamoto S, Yukata K, Shafi M, Akhundov K. Vascularized pedicled bone graft for
avascular necrosis of the capitate: case report. J Hand Surg Am. 2009 Sep;34(7):1303-7. doi:
10.1016/j.jhsa.2009.04.012. Epub 2009 Jun 4. PubMed

Peters SJ, Degreef I, De Smet L. Avascular necrosis of the capitate: report of six cases and review of the
literature. J Hand Surg Eur Vol. 2015 Jun;40(5):520-5. doi: 10.1177/1753193414524876. Epub 2014
Feb 25. PubMed

Question 94 of 100
The radiographs shown in Figures 1 and 2 reveal squamous cell carcinoma of the thumb involving the distal
phalanx. Following biopsy confirmation, what would be the most appropriate course of management?

Figure 1 Figure 2

A. Curettage and bone grafting


B. External beam radiation
C. Ray amputation of the thumb
D. Interphalangeal (IP) joint disarticulation

Correct answer: D

 Discussion

Squamous cell carcinoma of the fingertip/nail region is uncommon but remains the most common
malignancy in the hand. A high degree of suspicion is needed to diagnose this condition. Biopsy and
radiographs are necessary initially. The subsequent treatment depends on the extent of the lesion at the
time of presentation. Treatment can vary from Mohs micrographic surgery (MMS) to digital amputation.

Amputation is recommended when bone involvement is present. In this patient, because the distal
phalanx tip is involved and no further bone involvement proximally was observed, an amputation at the
IP joint level is recommended. More proximal involvement would require a more proximal amputation
level.

Curettage and bone graft is not appropriate for this malignant lesion. External beam radiation therapy is
not a first-line treatment option for this condition. Metastatic spread is uncommon. MMS is
inappropriate when bone invasion has occurred.
 Recommended Readings

English C, Hammert WC. Cutaneous malignancies of the upper extremity. J Hand Surg Am. 2012
Feb;37(2):367-77. doi: 10.1016/j.jhsa.2011.11.019. PubMed

Kakar S, Endress R. Skin cancer of the hand: current concepts. J Am Acad Orthop Surg. 2015
May;23(5):307-16. doi: 10.5435/JAAOS-D-14-00040. Full text

Askari M, Kakar S, Moran SL. Squamous cell carcinoma of the hand: a 20-year review. J Hand Surg
Am. 2013 Nov;38(11):2124-33. doi: 10.1016/j.jhsa.2013.08.090. Epub 2013 Sep 17. PubMed

Plate AM, Steiner G, Posner MA. Malignant tumors of the hand and wrist. J Am Acad Orthop Surg.
2006 Nov;14(12):680-92. Full text

Question 95 of 100

Which examination finding points toward a brachial plexus injury rather than root avulsion?

A. Winging of the scapula


B. Intact rhomboid function
C. A biceps with 0/5 strength
D. An ipsilateral clavicle fracture

Correct answer: B

 Discussion

A brachial plexus injury distal to the root level should leave the rhomboid muscle with intact function.
Root avulsions of C5-6 will cause weakness of the rhomboids. The branching of the dorsal scapular
nerve is proximal and often spared with upper brachial plexus injuries. Winging and biceps weakness
may occur with either injury, and an ipsilateral fracture does not differentiate an avulsion from a
brachial plexus injury.
 Recommended Readings

Tubbs RS, Tyler-Kabara EC, Aikens AC, Martin JP, Weed LL, Salter EG, Oakes WJ. Surgical anatomy
of the dorsal scapular nerve. J Neurosurg. 2005 May;102(5):910-1. PubMed

Caporrino FA, Moreira L, Moraes VY, Belloti JC, Gomes dos Santos JB, Faloppa F. Brachial plexus
injuries: diagnosis performance and reliability of everyday tools. Hand Surg. 2014;19(1):7-11. doi:
10.1142/S0218810414500026. PubMed

Question 96 of 100

Figure 1 is the radiograph of an 18-year-old right-hand-dominant man who has pain and stiffness 3 months after
sustaining an injury to his dominant ring finger while playing basketball. An examination reveals significant
proximal interphalangeal (PIP) joint swelling with active and passive PIP joint motion of 15/40 degrees of flexion.
What is the best next step?

Figure 1

A. Supervised hand therapy


B. Hemi-hamate autograft
C. Dynamic external fixation
D. Open reduction and internal fixation (ORIF)
Correct answer: B

 Discussion

Figure 2 Figure 3

Figure 4 Figure 5

This patient has a subacute PIP joint dorsal fracture dislocation with involvement of 50% to 60% of the
palmar articular surface of the base of P2. A "V sign" (Figure 2) is evident, indicating dorsal subluxation
of the joint. In some cases, an ORIF is possible, but substantial comminution often precludes proper
restoration of the critical volar buttress. Therapy is not the answer because the joint is dorsally
subluxated and must be corrected. Dynamic external fixation on its own would not result in a reduced
joint. The hemi-hamate autograft has proven useful in this type of scenario and serves to restore the
volar buttress of P2 using an osteochondral autograft harvested from the distal articular aspect of the
hamate at its articulation with the fourth/fifth metacarpal bases. Intraoperative clinical photographs and a
postsurgical radiograph are shown in Figure 3 through 5.
 Recommended Readings

Williams RM, Kiefhaber TR, Sommerkamp TG, Stern PJ. Treatment of unstable dorsal proximal
interphalangeal fracture/dislocations using a hemi-hamate autograft. J Hand Surg Am. 2003
Sep;28(5):856-65. PubMed

Williams RM, Hastings H 2nd, Kiefhaber TR. PIP Fracture/Dislocation Treatment Technique: Use of a
Hemi-Hamate Resurfacing Arthroplasty. Tech Hand Up Extrem Surg. 2002 Dec;6(4):185-92. PubMed

Frueh FS, Calcagni M, Lindenblatt N. The hemi-hamate autograft arthroplasty in proximal


interphalangeal joint reconstruction: a systematic review. J Hand Surg Eur Vol. 2015 Jan;40(1):24-32.
doi: 10.1177/1753193414554356. Epub 2014 Oct 22. Review. PubMed

Calfee RP, Kiefhaber TR, Sommerkamp TG, Stern PJ. Hemi-hamate arthroplasty provides functional
reconstruction of acute and chronic proximal interphalangeal fracture-dislocations. J Hand Surg Am.
2009 Sep;34(7):1232-41. doi: 10.1016/j.jhsa.2009.04.027. PubMed

Question 97 of 100

The examination finding shown in Video 1 is consistent with which defect?

Video 1

A. Trigger finger
B. Flexor digitorum profundus (FDP) incompetence
C. Flexor digitorum sublimis (FDS) incompetence
D. Extensor digitorum communis (EDC) incompetence

Correct answer: B

 Discussion

The video shows the lack of tenodesis caused by the incompetence of the FDP tendon to the ring finger,
which can be attributable to a laceration, tendon rupture, or avulsion. Note how the ring finger stays
extended (compared to the other digits) when the extensor tendons are tightened during wrist extension.
The other fingers are pulled into flexion by the FDP tendons when the extensor tendons are relaxed
during wrist extension. With the wrist flexed, the extensor mechanism to all fingers appears to be
functioning normally. Findings indicating a trigger finger would be locking in flexion of the proximal
interphalangeal joint. FDS incompetence can only be detected by blocking FDP function of the other
fingers and actively flexing the examined finger.

 Recommended Readings

Wolfe SW, Pederson WC, Hotchkiss RN, Kozin SH, eds. Green’s Operative Hand Surgery, 6th ed.
Philadelphia, PA: Churchill Livingstone Elsevier; 2011:189.

Strickland JW. Flexor Tendon Injuries: I. Foundations of Treatment. J Am Acad Orthop Surg. 1995
Jan;3(1):44-54. Full text

Question 98 of 100

Compared with percutaneous pinning with Kirschner wires (K-wires), the treatment of metacarpal neck fractures
with cannulated intramedullary screws is associated with

A. increased rates of soft-tissue infection.


B. greater initial construct stiffness and peak load until failure.
C. a slower return of digital range of motion.
D. an earlier time to bony union.
Correct answer: B

 Discussion

In a biomechanical study, headless compression screws showed superior load to failure, higher three-
point bending strength, and greater strength in axial loading compared with percutaneous K-wire
fixation for metacarpal neck fractures. Headless compression screws provide greater initial stability to
allow earlier motion in the postoperative period. No data comparing infection rates between the two
methods of fixation are available; however, it is assumed that K-wires placed outside of the skin would
have increased rates of infection. Neither fixation method would increase the time to healing.

 Recommended Readings

Jones CM, Padegimas EM, Weikert N, Greulich S, Ilyas AM, Siegler S. Headless Screw Fixation of
Metacarpal Neck Fractures: A Mechanical Comparative Analysis. Hand (N Y). 2017 Sep
1:1558944717731859. doi: 10.1177/1558944717731859. [Epub ahead of print] PubMed

Doarn MC, Nydick JA, Williams BD, Garcia MJ. Retrograde headless intramedullary screw fixation for
displaced fifth metacarpal neck and shaft fractures: short term results. Hand (N Y). 2015 Jun;10(2):314-
8. doi: 10.1007/s11552-014-9620-3. PubMed

Avery DM 3rd, Klinge S, Dyrna F, Pauzenberger L, Lam D, Cote M, DiVenere J, Obopilwe E,


Mazzocca A, Rodner C. Headless Compression Screw Versus Kirschner Wire Fixation for Metacarpal
Neck Fractures: A Biomechanical Study. J Hand Surg Am. 2017 May;42(5):392.e1-392.e6. doi:
10.1016/j.jhsa.2017.02.013. Epub 2017 Mar 27. PubMed

Question 99 of 100

Figures 1 through 4 show the radiographs and MRI obtained from a 40-year-old man who has a 6-week history
of ring finger pain, redness, and swelling after puncturing the finger with a toothpick. Purulent drainage from
the puncture wound site grew Eikenella corrodens. The patient was initially treated with oral antibiotics for 10
days and then intravenous (IV) antibiotics for 3 weeks. What is the best next step in treatment?
Figure 1 Figure 2
Figure 3 Figure 4

A. Continued IV antibiotics for 4 weeks


B. Continued oral antibiotics for 6 weeks
C. Bone scan, biopsy, and metastatic work-up
D. Surgical débridement along with antibiotics

Correct answer: D
 Discussion

This patient has a septic distal interphalangeal joint, which was treated with antibiotics alone. As a
result, the patient developed osteomyelitis with bone destruction and abscess. The best way to treat this
problem is to perform surgical débridement of bone and soft tissue, along with abscess drainage and an
appropriate antibiotic regimen. Antibiotic treatment without surgery would not be successful in
eliminating this particular infection. Bone scan with biopsy is not the correct option, because this
problem is an infection and not a tumor, and MRI already has provided enough diagnostic information.

 Recommended Readings

Reilly KE, Linz JC, Stern PJ, Giza E, Wyrick JD. Osteomyelitis of the tubular bones of the hand. J
Hand Surg Am. 1997 Jul;22(4):644-9. PubMed

Rayan GM, Putnam JL, Cahill SL, Flournoy DJ. Eikenella corrodens in human mouth flora. J Hand
Surg Am. 1988 Nov;13(6):953-6. PubMed

Robinson LG, Kourtis AP. Tale of a toothpick: Eikenella corrodens osteomyelitis. Infection. 2000
Sep;28(5):332-3. PubMed

Question 100 of 100

A 63-year-old woman is seen 10 weeks after sustaining a closed minimally displaced distal radius fracture. She
has been in a short-arm cast and reports minimal pain but notes that she is having difficulty using her thumb. An
extensor pollicis longus (EPL) tendon rupture is suspected. Which examination finding would confirm lack of EPL
function?

A. Positive froment sign with the ulnar palm flat on a table


B. Weak thumb abduction with the dorsal palm flat on a table
C. Inability to flex the thumb with the palm flat on a table
D. Inability to extend the thumb with the palm flat on a table
Correct answer: D

 Discussion

As many as to 5% of patients with a nondisplaced distal radius fracture experience EPL rupture. The
extensor pollicis brevis (EPB) tendon often attaches to the extensor hood and sometimes continues more
distally, providing weak metacarpophalangeal extension even in the setting of EPL disruption. However,
because of the vector of its pull, the EPB cannot extend the thumb dorsal to the plane of the palm. A
positive Froment sign is noted when flexion of the thumb interphalangeal joint with an attempted key
pinch is caused by adductor pollicis weakness from ulnar nerve dysfunction. Compression of the median
nerve in the carpal tunnel affects the recurrent motor branch of the abductor pollicis brevis, leading to
thenar atrophy. The flexor pollicis longus tendon (FPL) is intact so the patient would not have difficulty
flexing the thumb with the palm flat.

 Recommended Readings

Kulshreshtha R, Patel S, Arya AP, Hall S, Compson JP. Variations of the extensor pollicis brevis tendon
and its insertion: a study of 44 cadaveric hands. J Hand Surg Eur Vol. 2007 Oct;32(5):550-3. Epub 2007
Jun 12. PubMed

Roth KM, Blazar PE, Earp BE, Han R, Leung A. Incidence of extensor pollicis longus tendon rupture
after nondisplaced distal radius fractures. J Hand Surg Am. 2012 May;37(5):942-7. doi:
10.1016/j.jhsa.2012.02.006. Epub 2012 Mar 29. PubMed

Best of luck

You might also like